Pediatric Ophth/

Question 1 of 130 On occasion, it may be difficult to distinguish a moderate to severe abducens nerve palsy from Duane's syndrome. Which of the following is more characteristic of an abducens nerve palsy than of Duane's syndrome?

Diplopia

Esodeviation of 20 prism diopters in primary gaze

Abnormal forced duction to abduction

Palpebral fissure narrowing on adduction Please select an answer Feedback: Duane's syndrome is a congenital abnormality characterized chiefly by a disturbance in ocular motility. Because it is congenital, is rarely elicited. Classically, abduction is limited, with tightness of the medial rectus muscle on forced ductions. The palpebral fissure often narrows ( retraction) on attempted adduction. Duane's syndrome typically demonstrates a small to moderate esodeviation in primary gaze. An abducens nerve palsy with a significant limitation on abduction almost always manifests a large esodeviation in primary gaze.

Question 2 of 130 Which of the following statements about is least likely?

Infants with infantile who cross-fixate may have amblyopia.

Amblyopia in the presence of latent responds poorly to occlusion therapy.

Patients with intermittent commonly develop amblyopia.

Strabismic amblyopia is more likely than deprivational amblyopia to respond to occlusion. Please select an answer Feedback: Contrary to popular belief, infants with infantile esotropia may crossfixate and still have significant amblyopia. For example, a patient may prefer to fixate with the right eye until brought into extreme right gaze, at which time the patient switches to the left eye. The patient is a cross-fixator, but has amblyopia in the left eye. If no amblyopia is present, fixation should change from one eye to the other in the primary position of gaze. Amblyopia due to visual deprivation (ie, , corneal cloudiness) is almost always less responsive to occlusion treatment than amblyopia due to strabismus. It is not uncommon for patients with intermittent exotropia to have amblyopia, although the amblyopia is usually mild. Latent nystagmus is not a contraindication to amblyopia treatment; in fact, patients with latent nystagmus do respond to occlusion therapy.

Question 3 of 130 Which of the following methods is not used to detect ocular misalignment?

Cover/uncover test

Amblyoscope

Retinoscopy

Bruckner test Please select an answer Feedback: Ocular misalignment can be detected by a number of methods. The simplest and least accurate is the light reflex (Hirschberg) test. A more accurate method is the cover/uncover test. The Bruckner test compares the intensity of the light reflex between the eyes. In the presence of strabismus, the deviating eye has a brighter reflex. Retinoscopy may detect irregularities in the visual axis and refractive errors, but is not useful in determining the presence of strabismus.

Question 4 of 130 You previously saw a 5-year-old who was referred by school screening with visual acuities of 20/20 OD and 20/60 OS as using the E game (single letter). Cover testing revealed no deviation at distance or near. Worth 4-dot testing showed fusion at one-third meter and suppression at 6 m. Cycloplegic refraction was +1.50 sph OD and +3.75 +1.50 x 90 OS. Vision was not improved in either eye. The rest of the examination was normal. The child is treated with glasses and one month of occlusion OD. Examination now reveals a visual acuity of 20/20 OD and 20/25 OS and a left esotropia of 40 prism diopters at distance and near. What is the best alternative at this time?

Full-time occlusion OD

Half-time occlusion OD

Rx plano OD, +2.25 +1.50 x 90 OS

Rx +1.50 sph OD, +3.75 +1.50 x 90 OS Please select an answer Feedback: The development of a secondary strabismus is an unfortunate, but not entirely unexpected, complication of occlusion in patients with anisometropic amblyopia and no previous strabismus. When patching is used, parents should always be warned of this possibility beforehand so as to avoid an unpleasant surprise. Because improvement in vision is of prime importance, the risk must be taken. With improved vision, recovery of binocular function is the next goal. Discontinuing all occlusion will result only in a recurrence of the amblyopia now, due to the suppression of diplopia secondary to strabismus. Reduction of occlusion to part-time may allow a chance for binocularity to be reestablished and may maintain vision, but the accommodative element of the undercorrected preferred eye will continue to drive the left eye into a convergent position. For this reason, prescription of the full hyperopic correction is now indicated to reduce all accommodative elements of the strabismus. If the eyes do not straighten spontaneously with the new lenses, Fresnel prisms may be used to restore binocularity, and then gradually discontinued. Vision OS should be monitored carefully, and further part-time occlusion should be considered if acuity begins to fall. In the event the strabismus is not cured by prisms, surgical intervention will quite likely be necessary.

Question 5 of 130 A 7-year-old girl with an accommodative esotropia has been corrected with a +4.50 sph and +3.00 bifocals for the past 3 years. She demonstrates 3 prism diopters (PD) of on a distance target. On a near target, the esotropia is 30 PD through the distance correction, and the esophoria is 5 PD through the +3.00 bifocal. At near with a +2.00 add, she has an 8 PD esophoria, with a +1.00 add 12 PD esotropia. Through bifocals, she has 70 seconds of stereoacuity. What is the best next step in her management?

Reduce the distance correction

Reduce the bifocal add

Start divergence exercises

Maintain with present eyeglasses Please select an answer Feedback: It is unlikely this child will be able to eliminate eyeglasses completely. Furthermore, reducing the distance correction makes it less likely she would be able to reduce the bifocal. Most ophthalmologists would begin to wean her of the bifocal and allow her to slowly build her divergence amplitudes

Question 6 of 130 Which of the following statements is not accurate about the prism alternate cover test?

Stacking of a 30 and a 40 prism diopter base-out prism will give an additive effect of approximately 70 PD.

Plastic prisms should be held in the frontal plane position to give an accurate measurement.

A vertical prism and a horizontal prism can be stacked to give a good estimate of the horizontal and vertical deviation.

High plus lenses cause the measured deviation to be less than the true deviation. Please select an answer Feedback: Prisms do not add linearly when stacked together in the same direction. Although one prism surface may be correctly positioned, the other will always be misaligned, giving a much greater effect than intended. If a 30 and a 40 prism diopter plastic prism are stacked together in the same direction with the back surface placed in the frontal plane, the actual amount of prism displacement will be 141 PD. This additive phenomenon does not occur if a horizontal prism and a vertical prism are stacked. Plastic prisms are best positioned with the back surface parallel with the frontal plane of the patient (frontal plane position). High plus lenses result in the measured deviation being smaller than the true deviation, due to the prismatic effect induced by a high-powered in the presence of strabismus.

Question 7 of 130 A 3-year-old is found to have a of +3.00 +0.75 X 90 in both eyes. The child is given the full cycloplegic correction and returns with a residual deviation of 15 esotropia at distance and 25 esotropia at near. A second refraction reveals no change in the refractive error. What is the single best treatment now?

Surgery

+4.00 +0.75 x 90 for both eyes

Miotics

Bifocal eyeglasses Please select an answer Feedback: If two cycloplegic refractions reveal no change, it is unlikely that a miotic will have any effect on the esodeviation. Additional hyperopic correction is likely to result in blurred vision, which will discourage the child from wearing the glasses. If the distance deviation cannot be reduced to a range compatible with monofixation (8 prism diopters or less), the addition of a bifocal would serve no useful purpose. The best choice is surgery.

Question 8 of 130 Which of the following findings is not consistent with unilateral amblyopia?

Stable or improved vision with a neutral density filter

A small relative afferent pupillary defect

Strabismus

Central on a Goldmann visual field Please select an answer Feedback: and strabismus are leading causes of unilateral amblyopia. Patients with amblyopia usually have better scotopic vision, as exemplified by improved acuity when tested with a neutral density filter. A small relative afferent pupillary defect is fairly common in amblyopia. A central depression in the visual field has been described, but is extremely difficult to detect with normal-sized test targets. The presence of a central scotoma on a standard visual field test should suggest another cause.

Question 9 of 130 A 5-year-old is referred by school screening with visual acuities of 20/20 OD and 20/60 OS using the E game (single letter). Cover testing reveals no deviation at distance or near. Worth 4-dot testing shows fusion at one-third meter and suppression at 6 m. Cycloplegic refraction is +1.50 sph OD and +3.75 +1.50 x 90 OS. Vision is not improved in either eye. The rest of the examination is normal. What should the first treatment option be?

Part-time occlusion OD

Rx plano OD, +2.25 +1.50 x 90 OS

Full-time occlusion OD

Rx +1.50 sph OD, +3.75 +1.50 x 90 OS Please select an answer Feedback: This child presents with a significant anisometropic amblyopia as evidenced by the fact that the decreased vision is not improved with the proper refraction. At age 5, the child may recover vision either by occlusion of the preferred right eye, or perhaps by correction of the refractive error alone. In this case, the prescription of eyeglasses to correct the full astigmatic error and the difference between the preferred eye and the amblyopic eye is the preferred response. One problem with giving the full cycloplegic refraction is that the child may object to the full correction in front of the preferred eye and reject the eyeglasses entirely. Not correcting the full astigmatic error may lead to some improvement of vision but cannot be expected to give the same quality of vision OS as as that of OD, thus continuing the ocular preference OD. Occlusion of OD may well be necessary if vision OS does not improve with eyeglasses; however, occlusion is not without the danger of induced loss of binocularity and a resulting secondary strabismus. This child continues to show some peripheral fusion on the Worth 4-dot test and has the definite potential for spontaneous improvement of vision with eyeglasses alone.

Question 10 of 130 Which of the following is not commonly associated with ?

Myelinated nerve fibers

Leber's congenital amaurosis

Retinopathy of prematurity

Neonatal vitreous hemorrhages Please select an answer Feedback: of prematurity, myelinated nerve fibers, and neonatal vitreous hemorrhages have all been associated with myopia in infants and young children. The mechanism for this association is unknown, although it is thought that early and asymmetric visual deprivation may be a factor. Leber's congenital amaurosis is commonly associated with high hyperopia.

Question 11 of 130 A 4-month-old is referred because of jerky eye movements noted for the past 6 weeks. What is the least likely diagnosis?

Delayed visual maturation

Ocular albinism

Congenital stationary night blindness

Leber's congenital amaurosis Please select an answer Feedback: Congenital nystagmus is commonly associated with abnormalities of the visual sensory pathways. Ocular conditions that result in nystagmus include ocular albinism, Leber's congenital amaurosis, congenital stationary night blindness, and hypoplasia. Delayed visual maturation is an ill-defined entity that includes poorer visual responses in infants than are normal for their age, but without apparent cause. The infants usually do not have other specific neurologic abnormalities, although mild to moderate developmental delay does occur. Pattern visually evoked potentials have been described as normal for the age; however, this finding remains controversial. Nystagmus is not present with delayed visual maturation. Visual responses improve to normal or near normal with time.

Question 12 of 130 A 4-year-old girl has a 2-year history of wandering eyes when she is tired or ill. Her mother reports that the child tends to close her left eye when outdoors. Visual acuity testing using the linear E game reveals 20/20 OD and 20/40 OS. A cycloplegic refraction shows a refractive error of +2.00 +0.50 x 90 in each eye. The remainder of the eye examination is normal, with the exception of the motility and sensory examinations. Nonsurgical treatment of the condition diagnosed would not include which of the following?

Minus-lens therapy

Occlusion

Miotics

Orthoptic exercises Please select an answer Feedback: No one best treatment for intermittent exotropia exists. Most ophthalmologists recommend surgical correction for large deviations with poor control. For smaller deviations, exercises, minus lenses, occlusion, and prisms have all been effective, at least temporarily in selected patients. No definitive data is available that suggest which form of treatment is best for which patients. Miotics are useful in accommodative esodeviations, but are likely to make an intermittent exotropia worse.

Question 13 of 130 Abnormal head positions are a common finding with strabismic disorders. Other ocular, but nonstrabismic causes for an abnormal head position would not include which of the following?

Refractive errors

Nystagmus with a null point

Macular dystopia

Ectopia lentis Please select an answer Feedback: Abnormal head positions may be caused by a variety of ocular disorders other than strabismus. The most common cause is probably nystagmus associated with an attempt to position the eyes where there is the least movement (null point). Significant refractive errors, especially with , may also manifest as a head tilt or turn. In the presence of , an abnormal head position may help to shift the lens so as to place the visual axis in the phakic or aphakic region and achieve better vision. Macular dystopia will cause a disparity in the visual and optical axis of the eye, but alone should not result in an abnormal head position.

Question 14 of 130 A 5-year-old girl complains of diplopia 1 week after surgery for exotropia. Visual acuity is 20/30 OU. Measurements reveal an esotropia of 15 prism diopters at distance and at near. About 1 week later, the patient no longer complains of diplopia. Vision is now 20/20 OD and 20/40 OS. A left esotropia at distance is 25 prism diopters and at near is 30 PD. What should the next choice of treatment be?

Full-time occlusion of the right eye

Eyeglasses with prisms to correct the distance deviation only

Eyeglasses with +3.00 bifocals to correct the hypermetropia

Observation Please select an answer Feedback: The patient has shown several responses that merit concern: (1) a difference in visual acuity between the eyes not seen at the time of the last examination, (2) a significantly increased angle of esotropia at both distance and near, and (3) apparent suppression of binocular function, as she no longer reports any diplopia. In a 5-year-old child, these responses can lead to a loss of vision and binocularity that may be difficult to regain. Immediate institution of occlusion to regain vision and reduce the need for suppression is indicated. Other measures that do not restore vision, such as miotics or eyeglasses, may allow a reduction in the apparent angle of the deviation, but will not reduce suppression or improve vision if the esotropia persists. Prisms may allow peripheral fusion with loss of the need to suppress, but are not likely to restore vision in the amblyopic eye. If further vision is lost, binocular function may also be lost. The possibility of a blind-spot mechanism should always be considered in these cases. The apparent increase in the esotropia may be due to the patient reverting to accommodative convergence to rid herself of diplopia, as she could do before surgery. Occluding the right eye may not only increase vision in the left eye but also decrease the esotropic angle.

Question 15 of 130 Which of the following is not a common finding with ocular albinism?

Optic nerve atrophy

High refractive errors

Strabismus

Macular hypoplasia Please select an answer Feedback: Ocular albinism is primarily characterized by decreased pigmentation of the . This is manifested by diffuse transillumination defects and decreased pigment in the . Macular hypoplasia, strabismus, and high refractive errors are all associated with this disorder. Optic nerve atrophy is not a common feature.

Question 16 of 130 About 6 months after head trauma, a patient complains of image tilting that is worse with attempted reading and results in constant diplopia. The ocular motility examination reveals no vertical deviation, 5° of left excyclotorsion in primary gaze, and 12° in down gaze. What is the single best treatment?

A strengthening procedure on the anterior fibers of the left superior oblique muscle tendon

To encourage the patient to hold reading material higher

A weakening procedure on the left inferior oblique muscle

Eyeglasses with an oblique prism Please select an answer Feedback: Eyeglasses will not correct torsion. Weakening the left inferior oblique muscle is unlikely to affect the torsion in down gaze and may induce a vertical deviation. Because the problem is worse in down gaze, strengthening the anterior fibers of the left superior oblique tendon will selectively improve the rotating action of the superior oblique muscle without affecting its vertical and horizontal actions.

Question 17 of 130 What is a large A or V pattern usually associated with?

Duane's syndrome

Monocular elevation deficiency

Latent nystagmus

Oblique muscle dysfunction Please select an answer Feedback: Neither an A nor a V pattern is a common finding with Duane's syndrome, monocular elevation deficiency, or latent nystagmus. A large A or V pattern is almost always associated with abnormal overaction or underaction of one or more oblique muscles. Treatment for large patterns usually requires surgery on the oblique muscles.

Question 18 of 130 Which of the following clinical findings does not suggest a bilateral superior oblique palsy?

Excyclotorsion of 12°

V-pattern esotropia

Hyperdeviation in primary gaze of 15 prism diopters

Right on right tilt and left hypertropia on left tilt Please select an answer Feedback: Bilateral superior oblique palsies can be difficult to identify, especially when they are very asymmetric. Clues to bilateral involvement include an esodeviation that is greater in down gaze, a hyperdeviation that reverses (changes from right to left) on side gaze and with head tilts, and large degrees of excyclotorsion. Compared with a unilateral palsy, bilateral superior oblique palsies usually have little if any hyperdeviation in primary gaze.

Question 19 of 130 A 4-year-old girl has a 2-year history of wandering eyes when she is tired or ill. Her mother reports that the child tends to close her left eye when outdoors. Visual acuity testing using the linear E game reveals 20/20 OD and 20/40 OS. A cycloplegic refraction shows a refractive error of +2.00 +0.50 x 90 in each eye. The remainder of the eye examination is normal, with the exception of the motility and sensory examinations. The history of wandering eye in this otherwise normal child should suggest what diagnosis?

Duane's syndrome

Infantile esotropia

Pseudostrabismus

Intermittent exotropia Please select an answer Feedback: Thinking about the possible causes of a wandering eye before carrying out the ocular motor examination on a child can be helpful, because the child may not cooperate if cover tests need to be repeated. It is, therefore, wise to consider the possible findings before starting the examination to assure the best chance of getting the necessary information. The history of onset at age 2 years makes the diagnosis of an infantile-onset strabismus less likely. Large-angle infantile are usually obvious to parents and others soon after birth. Pseudostrabismus, the appearance of crossing due to wide epicanthal folds in toddlers with straight eyes, may suggest to the parents the onset of esotropia earlier than it actually occurred. The history of eye closure, especially in bright sunlight, intermittency with state of wakefulness, and apparent later age of onset, should make one suspicious that the wandering eyes noted in the history are most likely due to an intermittent exodeviation

Question 20 of 130 A 5-year-old girl complains of diplopia 1 week after surgery for exotropia. Visual acuity is 20/30 OU. Measurements reveal an esotropia of 15 prism diopters at distance and at near. What is the best option at this time?

Full correction of the hypermetropic refractive error

Observation

Full-time alternate occlusion

Eyeglasses with prisms to correct both the distance and the near deviations Please select an answer Feedback: The presence of diplopia and a small overcorrection (10 to 15 prism diopters) during the first postoperative weeks has been shown to lead to the best long-term postoperative result. Angles of deviation larger than 20 prism diopters might be an indication for either occlusion or miotic therapy at this time, but the near deviation is probably not significant this early after surgery. Prisms may be of great benefit later but are not indicated this early. It is important to pay careful attention to visual acuity in children in the amblyopic age range. A slight and equal reduction in vision following surgery is not unusual and is due to conjunctival swelling and reduced patient cooperation. Changes in refractive error immediately following strabismus surgery are common as well, and may persist for several months.

Question 21 of 130 A 5-year-old boy who had allogenic bone marrow transplantation 1 year ago for acute lymphocytic leukemia presents for routine eye examination. Which of the following is not an ocular complication of bone marrow transplantation?

Retinal hemorrhage

Iris heterochromia

Superficial punctate keratopathy

Cataract Please select an answer Feedback: Ocular complications from bone marrow transplantation are related to the severity of graft-versus-host disease, the total dose and duration of corticosteroids, and the level of immunologic deficiency. Graft-versus-host disease can lead to significant problems with dry eyes and severe conjunctival scarring. are thought to be related to the dose and duration of corticosteroids, although visually significant cataracts are uncommon. Cytomegalovirus (CMV) secondary to immunologic deficiency, especially with associated positive CMV cultures, may be seen in these children. Radiation retinopathy, which has been reported rarely, is characterized by retinal hemorrhages and cotton-wool spots and is thought to result from the combination of total body irradiation and chemotherapy. Iris heterochromia is not an ocular complication of the therapeutic management associated with bone marrow transplantation.

Question 22 of 130 The differential diagnosis of monocular nystagmus in a young child would not include which of the following conditions?

Posterior fossa tumors

Glioma of the optic chiasm

X-linked ocular albinism

Spasmus nutans Please select an answer Feedback: Monocular nystagmus should alert the ophthalmologist to a possibly serious ocular or neurologic condition. Monocular nystagmus has been associated with posterior fossa tumors and gliomas of the optic chiasm or nerve. Spasmus nutans often presents with a very asymmetric nystagmus, but it may be monocular. Ocular albinism affects both eyes and is associated with bilateral nystagmus.

Question 23 of 130 A 29-year-old woman has a history of esotropia that was diagnosed at 4 years of age and treated with glasses and patching for a few months. She has had decreased vision in her right eye for as long as she can remember. She has noticed more crossing of her right eye in the past few years and would like it corrected. Her best-corrected visual acuity is 20/100 OD with a -0.50 sphere and 20/20 OS with a +0.50 sphere. Your examination reveals a 20-prism- diopter right esotropia in primary and side gaze. The esotropia decreases to 5 prism diopters in up gaze and increases to 30 prism diopters in down gaze. The remainder of her ocular examination is normal. Which of the following would be the best procedure to correct this deviation?

Resection of the right lateral rectus muscle and recession of the right medial rectus muscle (R and R OD)

R and R OD and a weakening procedure on the right inferior oblique muscle

R and R OD with infraplacement of the medial rectus muscle and supraplacement of the lateral rectus muscle

Bilateral recessions of the medial rectus muscles with infraplacement Please select an answer Feedback: This patient demonstrates a V-pattern esotropia and decreased visual acuity in the right eye, presumably resulting from amblyopia. The esotropia could be corrected with a recession and resection procedure of either eye or bilateral medial rectus recession. Because the patient has significant , it is reasonable to restrict surgery to the poorer eye if possible. Correction of the V pattern can best be achieved by vertical displacement of the horizontal rectus muscles. In this case, the medial rectus should be infraplaced and the lateral rectus supraplaced. An inferior oblique muscle recession is indicated only if the inferior oblique muscle is overacting and there is an associated incomitant vertical deviation.

Question 24 of 130

A 6-week-old infant was diagnosed with a left upper capillary hemangioma at birth. On examination, the patient is noted to have a large left upper eyelid capillary hemangioma and difficulty opening the left . This child is at risk for which of the following conditions?

Metastatic disease and death

Permanent levator damage in the left upper eyelid

Dense and irreversible amblyopia OS

Glaucoma OS Please select an answer Feedback: The child has an infantile capillary hemangioma that is causing complete obscuration of the visual axis of the left eye. The lesion is benign. The distribution of the lesion approximates that of a port-wine stain, which could be associated with Sturge-Weber syndrome (encephalorntrigeminal angiomatosis) and . The appearance, however, is most consistent with an infantile capillary hemangioma. If left untreated, a capillary hemangioma will almost always regress over several years. In this case, however, the lesion obscures the visual axis and is likely to result in dense and irreversible amblyopia if not treated within the first few months of life. Systemic or intralesional corticosteroids have proven effective in "shrinking" capillary hemangiomas in these situations, although there are potentially serious side effects from this form of therapy. Systemic or intralesional corticosteroids in combination with occlusion therapy of the right eye would be the most reasonable treatment for this patient.

Question 25 of 130 Deafness has not been commonly associated with which of the following ocular findings?

Anterior lenticonus

Retinal coloboma

Interstitial

Retinitis pigmentosa Please select an answer Feedback: A number of ocular conditions, both congenital and acquired, have been associated with neurosensory deafness. Classically described syndromes associated with deafness and their ocular findings include Usher's (), Alport's (anterior lenticonus), and Cogan's (interstitial keratitis) syndromes. Ocular colobomas are not commonly associated with neurosensory deafness.

Question 26 of 130 A pediatrician calls to refer a 3-year-old Caucasian boy with a 1 week history of esotropia, abduction limitation, and face turn. There is no history of ocular disease or surgery. The patient is brought in for examination 3 days after the initial phone consultation. The parents have noted a slight improvement in the crossed eyes. The child has no history of trauma but did have a flu-like illness 1 week prior to onset of the esotropia. The ocular examination is normal. The workup at this time should not include which of the following?

Erythrocyte sedimentation rate

MRI

Neurology referral

Observation Please select an answer Feedback: In childhood, almost 70% of abducens palsies can be accounted for by trauma, primary intracranial tumors (especially pontine gliomas and cerebellar astrocytomas), and congenital and postinfectious etiologies. Complete neurologic examination is appropriate to identify additional abnormalities that might suggest intracranial pathology. If other abnormalities are found, MRI should be considered to look for a tumor. The most likely cause in this case is a sixth-nerve palsy secondary to viral infection. Since the patient is improving, continued observation is reasonable. An erythrocyte sedimentation rate (ESR) would not be helpful in this case. The ESR might be elevated as a result of the recent viral illness. More importantly, systemic conditions associated with an elevated ESR are not related to the development of sixth-nerve palsies in childhood.

Question 27 of 130 A 13-year-old boy presents with a 6-month history of headaches, blurred vision, and intermittent crossed diplopia at near. His symptoms occur almost every day and are worse when he is tired. Your examination reveals an uncorrected acuity in each eye of 20/20. On a distant accommodative target, he is orthophoric. On a target at 1/3 m, he has a 15-prism-diopter . His near point of is 8 diopters in each eye. His fusional convergence amplitudes are 16 prism diopters to break and 12 to recover at 1/3 m, and 4 prism diopters to break and 2 to recover on a target at 6 m. Cycloplegic refraction reveals +0.75 sphere OD and +1.00 sphere OS. What would the best treatment option be?

Exercises for convergence insufficiency

Resection of both medial rectus muscles

Glasses with base-in prisms

Correction of the cycloplegic refractive error Please select an answer Feedback: The best treatment option for this patient would be exercises for convergence insufficiency. Many motivated patients with symptomatic convergence insufficiency will improve with exercises if their symptoms are related to near work. A base-in prism may be helpful in selected patients, although the patient may compensate for the prism and increase the size of the deviation. Prisms are probably best reserved for elderly patients whose accommodation cannot be used to aid convergence and for patients unwilling to do exercises. Correction of the hyperopia is likely to decrease convergence and aggravate the symptoms. Surgery can benefit patients with moderate-to-large deviations who have failed to respond to exercises and prisms. A persistent esodeviation with diplopia is common following strabismus surgery for convergence insufficiency.

Question 28 of 130 Optic nerve hypoplasia has not been commonly associated with which of the following characteristics?

Absence of the septum pellucidum

Hormonal dysfunction

High myopia

Paradoxic Please select an answer Feedback: Optic nerve hypoplasia may be unilateral or bilateral and may involve the entire disc or only a segment. Vision is usually affected, but the severity is highly variable. In addition to the visual limitations, patients with optic nerve hypoplasia, especially with bilateral involvement, have a greater than normal risk of midline brain abnormalities and hormonal dysfunction. One of the clinical signs occasionally found in this condition is a paradoxic pupil.

Question 29 of 130 A 7-year-old boy presents with a 9-month history of frequent episodes of blinking. The episodes are short in duration and occur randomly. Visual acuity is normal. The child's refractive error is +0.50 sphere in each eye. The remainder of the ocular examination is normal. Which of the following statements best describes this situation?

A conjunctival scraping to look for eosinophils and mast cells is indicated.

The patient is likely to have a tic disorder.

The blinking is most likely a habit.

A tear secretion test with and without anesthetic is likely to be diagnostic. Please select an answer Feedback: This patient most likely has a tic disorder. Tic disorders represent a clinical spectrum from the mild, transient form seen in up to 15% of children to the potentially devastating Tourette's syndrome, for which prevalence estimates vary from 1:200 to 1:2000. Tic disorders are characterized by involuntary, sudden, rapid, brief, repetitive, stereotyped, purposeless movements or vocalizations. There is often a positive family history, and males are more likely to be affected than females. Other behavioral problems are frequently associated with tic disorders; obsessive- compulsive disorder and attention-deficit hyperactivity disorder are the most common. Dry eye syndromes are very rare in children. It is unlikely with an otherwise normal examination that this child would have decreased tear secretion. Allergic or vernal is more common; however, it is often seasonal and will usually demonstrate abnormalities on anterior segment examination.

Question 30 of 130 Which of the following statements best describes developmental dyslexia?

Dyslexia is not common in juvenile delinquents.

Dyslexics often have below-normal intelligence.

Dyslexics rarely have a positive family history for dyslexia.

Dyslexics often present to ophthalmologists with visual symptoms. Please select an answer Feedback: Dyslexia is estimated to affect up to 8% of school-age children. As many as 75% of juvenile delinquents are thought to be dyslexic or to have learning disabilities. Recent evidence suggests that males and females may be affected nearly equally. A positive family history for learning disorders is not uncommon. If one parent is affected, a child has a five to tenfold greater risk of have a reading disorder than a child of unaffected parents. Most dyslexics are of normal intelligence or even gifted. Since almost all dyslexics are visually normal, they frequently pass unsuspected through offices with vague complaints such as headaches with reading, blurry blackboards, words running together, or reading difficulties. Parental requests for routine checkups are often initiated by disappointing performance rather than by a desire for screening or preventive care, as is usually presumed. The value of a careful history and early recognition cannot be overestimated.

Question 31 of 130 A 4-month-old boy is brought in for evaluation because of visual inattention and nystagmus. His do not respond to light and he has a refractive error of +4.00 OU. Both fundi appear normal. What condition would this represent?

Leber's

X-linked congenital stationary night blindness

Cortical blindness

Leber's congenital amaurosis Please select an answer Feedback: Both Leber's congenital amaurosis and X-linked congenital stationary night blindness may be associated with nystagmus, decreased vision, and a normal-appearing fundus in infancy. X-linked congenital stationary night blindness is usually associated with a myopic refractive error, whereas 75% of children with Leber's congenital amaurosis have 3 diopters or more of hyperopia. The fundus may appear normal at a very early age, but arteriolar narrowing, pigmentary changes, and optic atrophy become evident with time. Nystagmus is usually not present in children with cortical blindness or Leber's optic neuropathy. In addition, pupillary responses should be normal with cortical blindness, and Leber's optic neuropathy is not seen in infants.

Question 32 of 130 A 4-month-old infant girl is noted by her parents to have the unusual appearance of her right eye showing a temporally located retrolental mass. The otherwise healthy child was born 3 weeks prematurely with a birth weight of 5 pounds 6 ounces. Your examination reveals a normal left eye. The right eye demonstrates shallowing of the anterior chamber and a clear crystalline lens, as well as the retrolental and retinal abnormalities, such as a retinal fold extending from the to the retinal periphery. Examinations of the parents and an older sibling are unremarkable. What is the most likely diagnosis?

Norrie's disease

Familial exudative vitreoretinopathy

Incontinentia pigmenti

Persistent hyperplastic primary vitreous Please select an answer Feedback: A funnel-shaped emanating from the optic nerve in an infant of normal birth weight is most consistent with a diagnosis of persistent hyperplastic primary vitreous (PHPV). PHPV is a predominantly unilateral congenital ocular anomaly characterized by incomplete atrophy of the fetal hyaloid vascular system. Commonly associated findings include microphthalmia, a retrolental fibrovascular mass, progressive lens swelling with cataract formation, and elongation of the ciliary processes. PHPV may present with involvement of the posterior segment of the eye only, characterized by tractional retinal folds and detachment. Other disorders to consider include familial exudative vitreoretinopathy (FEVR), Norrie's disease, incontinentia pigmenti, and retinopathy of prematurity. All of these conditions are usually bilateral. FEVR is an autosomal dominant condition with variable expressivity; examination of the parents or siblings will help in ruling it out. Most patients with FEVR are asymptomatic. Incontinentia pigmenti is inherited as an X-linked dominant disorder that is usually lethal in males. It has characteristic skin vesicles that cicatrize in infancy. Norrie's disease is a rare, X-linked, recessive condition characterized by progressive deafness and mental retardation after infancy. Female carriers are unaffected. Ocular manifestations include vitreous hemorrhages that lead to retinal detachment, , and . Norrie's disease can be confirmed in male patients by evaluation of the blood for the Norrie's gene.

Question 33 of 130 The earliest signs of malignant hyperthermia does not include which of the following?

Masseter muscle spasm

Tachycardia

Tachypnea

High fever Please select an answer Feedback: Although malignant hyperthermia takes its name from the profound increase in body temperature that may occur, sometimes rising in excess of 112°P (45°C), high fever occurs relatively late in the disorder. The pathogenesis is not well understood, but malignant hyperthermia is an abnormal reaction to anesthetic agents in which a hypermetabolic state ensues, leading to severe metabolic acidosis, cyanosis, coagulopathy, hyperkalemia, and muscle necrosis. The earliest signs are tachycardia, tachypnea, and spasm of the masseter muscle (trismus). Early diagnosis based on these signs, keyed to a high index of suspicion, may be life-saving. Careful questioning concerning family and past medical history of malignant hyperthermia is indicated in all preoperative patients, and, when responses are positive, appropriate preventive measures (such as confirmatory musclebiopsy, pretreatment with dantroline, and avoidance of triggering anesthetics) should be taken prior to and during anesthesia.

Question 34 of 130 A 5-year-old child with horizontal nystagmus sees 20/50 when her head is held straight but can see 20/25 by adopting a 45 degrees left head turn. The nystagmus is unchanged when fixing on a near target. Which surgical approach would be most likely to reduce her head turn?

Recess the right medial rectus and the left lateral rectus muscles. Resect the right lateral rectus and the left medial rectus muscles.

Recess the left medial rectus muscle and resect the left lateral rectus muscle.

Recess the left medial rectus and the right lateral rectus muscles. Resect the left lateral rectus and the right medial rectus muscles.

Recess both medial rectus muscles and resect both lateral rectus muscles. Please select an answer Feedback: This child's excellent vision, combined with her large head turn, are most consistent with idiopathic or motor nystagmus. Presumably, the patient is adopting a head turn to fixate in null position. One surgical approach would be to create an exotropia that would stimulate a nystagmus-dampening convergence. In this case, the absence of a null position with convergence would make it unlikely that creating an exotropia would improve her head turn. Recession and resection of the horizontal to move both eyes in the direction of the head turn is the treatment of choice. This procedure, originally described by Anderson and Kestenbaum, has undergone a number of variations to achieve the best head alignment while avoiding the development of strabismus. In this specific case, a recession of the left medial and right lateral rectus muscles with resection of the left lateral and right medial rectus muscles should achieve the desired goal.

Question 35 of 130 A 3-year-old boy with a history of congenital esotropia surgically corrected at age 12 months has visual acuity of 20/30 with both eyes open, but only 20/60 when each eye is tested monocularly using an occluder. What is the most likely reason for this discrepancy?

Latent nystagmus

Binocular summation

Crowding phenomenon

Bilateral amblyopia Please select an answer Feedback: Latent nystagmus is often found in patients with congenital esotropia, independent of whether or not the esotropia is corrected. It is induced, or exacerbated, if there is a manifest component, brought on by occluding one eye, as occurs in routine visual acuity testing. This often results in the measured monocular acuity being significantly worse than the acuity measured with both eyes open, since the nystagmus will degrade the measured vision to some degree. Monocular occlusion should not affect the acuity in cases of bilateral amblyopia or in amblyopic eyes demonstrating the crowding phenomenon. Although vision is often a little better with both eyes open (binocular summation) than with each eye measured separately, the difference is usually no more than 0.5 to 1 Snellen line. In the crowding phenomenon, characteristic of amblyopic eyes, monocular visual acuity is significantly better (frequently by 2-3 lines) when measured using an isolated optotype than when using a line of multiple letters. Poor cooperation may also be an issue in visual acuity testing in children, but it does not usually result in the same reduction in acuity in each eye, as seen in this patient.

Question 36 of 130 The parents of a 5-month-old boy bring him into your office concerned that he does not seem to see anything. He was the product of a full-term, uncomplicated pregnancy and had no neonatal problems. Developmental history reveals that he is rolling over and starting to sit unsupported. On your examination, the child does not fixate or follow but will blink at a bright light. Cycloplegic refraction reveals +3.00 sphere OD and +3.50 sphere OS. Pupils are round, equal in size and briskly reactive to light, without a relative afferent pupillary defect. He has full ductions and versions, an approximately 2011 exotropia, but no nystagmus or wandering eye movements. He also has clear media and a normal fundus. What does this child most likely have?

Delayed visual maturation

Foveal hypoplasia

Refractive amblyopia

Leber's congenital amaurosis Please select an answer Feedback: Infants with profound sensory visual loss, such as that caused by Leber's congenital amaurosis or foveal hypoplasia, should demonstrate sluggish pupillary reaction and nystagmus by 2 to 4 months of age. Infants with high refractive errors will still fixate on a light and on large objects that are up close, such as a face, but this child does not do so. Poor visual attention in an otherwise normal baby is the hallmark of delayed visual maturation. Often these infants perform very well when tested with preferential grating acuity tests, such as Teller acuity cards, despite their lack of interest in lights and faces. Delayed visual maturation is a diagnosis of exclusion, confirmed only in retrospect. Although most infants with this diagnosis will demonstrate normal visual behavior by 1 year of age without specific therapy, careful follow-up every 3-4 months is warranted until the child demonstrates normal visual behavior or additional findings become apparent.

Question 37 of 130 Which of the following statements does not accurately describe chlamydial conjunctivitis of the newborn?

Silver nitrate 1% solution provides prophylaxis.

Oral erythromycin should be prescribed.

It is commonly associated with pneumonitis.

It is the most common infectious cause of neonatal conjunctivitis. Please select an answer Feedback: The most common infectious cause of neonatal conjunctivitis is Chlamydia trachomatis. Up to 10% of asymptomatic pregnant women have been shown to be culture positive for this organism. Approximately 30% to 40% of children born to untreated affected mothers will be infected. Approximately 15% of newborns with conjunctivitis develop chlamydial pneumonitis. Because of possible systemic involvement, oral erythromycin is the treatment of choice. Either erythromycin or tetracycline, given topically within 1 hour after birth, significantly decreases the likelihood of developing chlamydial conjunctivitis. Silver nitrate has little or no effect in preventing this infection.

Question 38 of 130 Which one of the following complications of intralesional corticosteroid injection in adnexal and orbital capillary hemangiomas has not been reported?

Central retinal artery occlusion

Globe perforation

Overlying skin atrophy or ulceration

Growth retardation Please select an answer Feedback: Local injection of corticosteroids has sometimes been reported to result in skin atrophy or ulceration, which can be cosmetically noticeable. This complication is generally avoidable by injecting the drug deep within the tumor rather than near the skin surface. Despite the original optimism that local injection would reduce the systemic side effects associated with systemic use, suppression of the pituitary-adrenal axis with secondary growth retardation can still occur with local injection. Central retinal artery occlusion has been reported following local injection of corticosteroids, possibly due to a retrograde flow of particles in the arterial circulation. Globe perforation has not been reported during intralesional injection in this setting.

Question 39 of 130 A mother complains that her 6-year-old daughter has had increased wandering of her left eye over the past year. Your examination notes an intermittent exotropia of 35 prism diopters with fixation at distance and 5 prism diopters of exophoria with fixation at near. Her uncorrected visual acuity is 20/20 OD and 20/20 OS. After a discussion with the mother, you decide on surgical treatment. Which of the following tests would be least helpful in surgical management of this patient?

Cycloplegic refraction

+3.00 lens test

30-minute occlusion test

Worth four-dot test Please select an answer Feedback: This case illustrates an exodeviation with a distance-to-near disparity. It is not clear from the information provided whether this is the patient's basic deviation, unaffected by fusional or accommodative convergence amplitudes. The occlusion test consists of occluding one eye for a specified period of time (usually at least 30 minutes) and reassessing the distance and near deviations without allowing the patient to regain fusion. Theoretically, this test will eliminate fusional convergence that is otherwise not detected with a traditional alternate prism-cover test. The +3.00 lens test measures the near deviation with and without the addition of a +3.00 sphere in front of both eyes. This test should eliminate accommodative convergence. Although each test may influence the other, in general both tests provide information on the mechanism of the distance-to-near disparity, which may suggest possible management options. A refraction is mandatory in the evaluation of any strabismus case. For example, correction of an anisometropia may provide better control of the deviation. The Worth four-dot test provides the least useful information for the management of this patient. The fact that the patient has a phoria on near vision indicates the presence of fusion.

Question 40 of 130

A 4-month-old infant boy presents with of the right eye that has been present since shortly after birth. Mucoid discharge can be expressed from the puncta with external pressure over the right nasolacrimal sac. What would the most reasonable treatment be?

Probing of the nasolacrimal duct

Massage of the nasolacrimal sac

Observation

Irrigation of the nasolacrimal duct

Please select an answer

Feedback: Congenital obstruction of the nasolacrimal duct, or dacryostenosis, occurs in 1% to 6% of newborn infants. Most of the obstructions open spontaneously within the first 6 months of life. Thereafter, spontaneous resolution may still occur, but is much less likely. Massage of the nasolacrimal sac appears to increase the rate of resolution of the obstruction, as compared with no massage. If massage is unsuccessful in clearing the obstruction, then probing of the nasolacrimal duct should be considered by 1 year of age.

Question 41 of 130 A child with unilateral traumatic cataract presents for treatment 9 months after the initial injury. After successful cataract extraction, the child is found to have severe deprivation amblyopia. The best visual prognostic outcome would be expected if the child acquired the traumatic cataract at which of the following ages?

5 years

3 months

1 year

3 years Please select an answer Feedback: The older a child is a the time of deprivation the better chance there has been significant visual development, and thus lesser chance of irreversible amblyopia developing.

Question 42 of 130 What is the probability that a patient with sporadic, unilateral, solitary retinoblastoma actually carries a germ-line (heritable) mutation?

0%

50%

3%

15% Please select an answer Feedback: Essentially all patients with bilateral retinoblastoma carry an abnormal gene. This is a heritable germ-cell mutation present in all of the affected individuals' cells. The chance of passing the gene to each offspring is 50%. Of patients with unilateral retinoblastoma, 15% have the heritable germ-cell mutation and 85% have a sporadic somatic mutation; that is, a mutation of the retinoblastoma gene in one retinal cell. The somatic mutation cannot be passed on.

Question 43 of 130 A mother brings her firstborn child in for an eye examination at age 1 week. The boy has bilateral leukocoria. The mother reports that her brother had a white reflex in both eyes as an infant and underwent bilateral enucleations; he now has three normal children. The woman's sister has had normal eye examinations; she has no children. The woman's maternal uncle is blind bilaterally of an unknown cause; his two children are normal. The baby's grandparents all had normal eyes. What is the most likely diagnosis?

Persistent hyperplastic primary vitreous

Retinoblastoma

Congenital cataracts

Norrie's disease Please select an answer Feedback: In this family, only males are affected. Males never pass on the disease, but their sisters have affected sons. This is an example of X-linked recessive inheritance. Retinoblastoma is a heritable cause of leukocoria; however, transmission is autosomal dominant with reduced penetrance. It would be unusual for none of the affected individuals to have affected children, while clinically normal women consistently do. Congenital cataracts may be heritable, but are usually autosomal dominant. Persistent hyperplasia of the primary vitreous is usually unilateral and rarely familial. Norrie's disease is an X-linked recessive condition affecting only males. Affected boys are usually blind from birth, with leukocoria and microphthalmia. Bilateral retinal detachments are common, and phthisis often results. Approximately 35% develop deafness and mental retardation.

Question 44 of 130 A 5-year-old boy has an ophthalmologic eye examination after failing a preschool vision screening. The examination reveals bilaterally subluxated lenses, with one lens down and in, and the other up and out. The child is tall for his age, with long, thin fingers and hyperextensible finger joints. Both parents are normal and the family history is unremarkable. What is the most likely diagnosis?

Weill-Marchesani syndrome

Homocystinuria

Hyperlysinemia

Marfan's syndrome Please select an answer Feedback: Although an autosomal recessive condition such as homocystinuria may appear to be likely because neither parent is affected, Marfan's syndrome is the most common disorder causing bilateral subluxated lenses in an otherwise normal child. The prevalence of Marfan's syndrome is about 1 in 10,000 compared with 1 in 100,000 for homocystinuria. Marfan's syndrome is autosomal dominant, but expressivity may be extremely variable and parents should be examined for subtle signs. Also, 15% of cases are new mutations and are often associated with increased paternal age. Homocystinuria is routinely screened for at birth, but certain types may escape detection with some assays.

Question 45 of 130 A 5-year-old boy has an ophthalmologic eye examination after failing a preschool vision screening. The examination reveals bilaterally subluxated lenses, with one lens down and in, and the other up and out. The child is tall for his age, with long, thin fingers and hyperextensible finger joints. Both parents are normal and the family history is unremarkable. What is the best first step for visual rehabilitation?

Aphakic or phakic eyeglass correction based on retinoscopy refined subjectively

Contact lenses

Pars plana lensectomy

Lensectomy by phacoemulsification Please select an answer Feedback: Significant complications are associated with removing a subluxated lens, regardless of the method used. In the majority of cases, good vision can be obtained with eyeglasses or contact lenses if a careful refraction is performed. Refraction through the lens usually gives high myopic astigmatism, which progresses if the lens continues to dislocate. If the acuity is not normal with this correction, then a refraction through the aphakic space, no matter how small, will often improve vision to near normal. Retinoscopy and subjective refinement, when possible, should be done through both the phakic and the aphakic spaces.

Question 46 of 130 A 5-year-old boy has an ophthalmologic eye examination after failing a preschool vision screening. The examination reveals bilaterally subluxated lenses, with one lens down and in, and the other up and out. The child is tall for his age, with long, thin fingers and hyperextensible finger joints. Both parents are normal and the family history is unremarkable. Which of the following is the least helpful in the initial workup of the patient?

Referral to a geneticist

Echocardiogram

Serum homocystine levels

Electroretinogram Please select an answer Feedback: The differential diagnosis of isolated subluxated lenses in children includes trauma, Marfan's syndrome, syphilis, ectopia lentis et pupillae (autosomal recessive), autosomal dominant subluxated lenses, hyperlysinemia, sulfite oxidase deficiency, and homocystinuria. A careful history to rule out trauma should be followed by testing for the possible life-threatening condition Marfan's syndrome; an echocardiogram can rule out the characteristic aortic root dilation and/or mitral valve prolapse. Serum homocystine levels can detect homocystinuria so that early dietary treatment can be instituted to prevent serious complications. Patients with homocystinuria are at risk for life- threatening thrombosis during anesthesia. A full genetic workup may be indicated to uncover the rarer conditions that are often associated with other systemic findings. The electroretinogram gives no specific information to aid in determining the cause of subluxated lenses.

Question 47 of 130 A 3-year-old boy is found to have decreased vision secondary to bilateral amblyopia. Which of the following is the most likely finding on cycloplegic retinoscopy?

8 diopters of hyperopia in each eye

3 diopters of hyperopia in the right eye, and 5 diopters of hyperopia in the left eye

3 diopters of myopia in the right eye, and 5 diopters of myopia in the left eye

3 diopters of myopia in each eye Please select an answer Feedback: Moderate myopia will very rarely cause bilateral amblyopia, because the is stimulated at near even if assymetric. Marked hyperopia, however, can lead to significant amblyopia, because the visual system can be deprived at both near and far leading to poor visual development in both eyes.

Question 48 of 130 A characteristic type of abnormality of the crystalline lens would not be found in which of the following conditions?

Stickler's syndrome

Carrier state of Lowe's syndrome

Neurofibromatosis type II

Axenfeld's anomaly Please select an answer Feedback: Some syndromes or conditions have a greater than expected frequency of abnormalities of the crystalline lens that may provide a clue to their diagnosis. Stickler's syndrome is a dominantly inherited condition characterized by high myopia, retinal detachment, high arched palate, and a degenerative arthropathy. Cortical lens opacities were noted in patients in two studies. Posterior lens opacities occur with a greater than normal frequency in patients with neurofibromatosis type II. Finally, females who carry the X-linked gene for Lowe's syndrome (oculocerebrorenal syndrome) have been reported to have an increased likelihood of posterior polar and cortical lens opacities. Axenfeld's anomaly consists of prominent Schwalbe's ring and iris processes. Cataracts are not characteristically associated with this condition.

Question 49 of 130 Which of the following is not a common complication of retinopathy of prematurity?

Refractive error

Cataract

Strabismus

Retinal detachment Please select an answer Feedback: Infants with retinopathy of prematurity have a greater tendency than full-term infants to develop refractive errors and strabismus. Severe retinopathy of prematurity is commonly associated with retinal detachment and closed- angle glaucoma. Although lens opacities may occur as a consequence of stage V retinopathy of prematurity with phthisis, they are much less common than the other conditions.

Question 50 of 130

Which of the following is not an accepted treatment for retinoblastoma?

Enucleation

Cryotherapy

Photocoagulation

Transscleral diathermy

Please select an answer

Feedback: Small retinoblastomas can be destroyed using laser photocoagulation, cryotherapy, or external beam radiation. Enucleation is reserved for eyes with large tumors. Transscleral diathermy should not be used for malignant intraocular tumors, because it may promote extrascleral extension.

Question 51 of 130 A 3-year old boy presents for evaluation becuase his parents believe the child has intermittent esotropia. The examination is consistent with 15 prism diopters of intermittent esotropia at distance and near. Which of the following is the most appropriate initial therapy?

Spectacles for correction of hyperopia

Bilateral medial rectus muscle recession

Plano spectacles with base-out prisms

Schedule CT-scan of the brain Please select an answer Feedback: Accommodative esotropia is usually intermittent at onset, but may become constant. Accomodative esotropia presents between 6 months and 7 years of age with an average onset of 2 1/2 years of age. When uncorrected hyperopia is present, the patient is forced to accommodate to sharpen the retinal image leading to increased convergence and, in the absence of sufficient fusional divergence, eotropia results. Treatment consists of correction of the full amount of hyperopia determined under with cyclopentalate or atropine. A CT-scan of the brain is not the most appropriate initial therapy, and is usually not required if the child's intermittent esotopia is controlled with spectacles for full correction of hyperopia. Surgical correction may be required when a patient with accommodative esotropia fails to regain fusion with glasses, but is not an appropriate initial therapy. Base-out prisms are not needed, because fusion is regained once the patient begins wearing spectacles correcting the full amout of hyperopia.

Question 52 of 130 A 16-year-old boy has a history of normal vision in both eyes. He presents complaining of a sudden decrease in vision in the right eye. His visual acuity is 20/200 OD and 20/15 OS. A relative afferent pupillary defect is present in the right eye, and confrontation visual fields reveal a dense central scotoma OD. Fundus examination reveals a normal macula with a hyperemic-appearing disc OD. The result of MRI of the head and optic nerves is normal. If an abnormality in this patient's DNA related to visual loss were found, what would it most likely reveal?

A mutation in the rhodopsin gene on chromosome 3

Aa mutation of the mitochondrial DNA

A mutation on the X chromosome

Chromosome 13 deletion Please select an answer Feedback: This case is most consistent with a diagnosis of Leber's hereditary optic neuropathy. This condition is associated with mutations in the mitochondrial DNA in at least three loci: nucleotides 11778, 3460, and 14484. For reasons that are not well understood, males are affected more commonly than females. The disease is inherited from the maternal DNA, with a woman who harbors the mutation passing it to all of her offspring. Why only a fraction of the individuals who harbor the mutation will actually develop optic neuropathy is not currently understood. Molecular tests that reveal the presence of one of these three mutations are an extremely reliable way to make the diagnosis. The clinical presentation of Leber's hereditary optic neuropathy is not diagnostic of the condition. Neuroimaging should be considered in these patients at least once to rule out the possibility of a lesion of the optic nerve or brain. A mutation in the rhodopsin gene on chromosome 3 has been found in some types of retinitis pigmentosa. A mutation on the X chromosome has been associated with a number of ocular conditions, although not with Leber's hereditary optic neuropathy. Ocular manifestations of a chromosome 13 deletion include microphthalmia, colobomata, and retinoblastoma.

Question 53 of 130 Cytomegalovirus (CMV) is the most common intrauterine infection and has been isolated from the urine in up to 3% of newborns. A small percentage of these infants show clinical symptoms in the neonatal period and have widespread congenital anomalies. Which of the following is not a typical congenital anomaly associated with intrauterine CMV infection?

Chorioretinitis

Hepatosplenomegaly

Microphthalmia

Limb anomalies Please select an answer Feedback: The predominant abnormalities associated with CMV infection are related to damage to the central nervous system. Among the abnormalities noted are mental retardation, seizures, spasticity, and deafness. Other manifestations include premature birth, small size for gestational age, microcephaly, , , thrombocytopenia, and anemia. Reported ocular findings include , microphthalmia, cataracts, and optic disc anomalies. Limb anomalies are not generally associated with intrauterine CMV infection.

Question 54 of 130 A 3-month-old infant has a history of a difficult birth with prolonged hypoxia. The child is noted to have severe neurologic impairment, including poor visual response. The ocular examination demonstrates no apparent visual fixation responses. Wandering ocular movements are present, but are not typical of nystagmus. The remainder of the examination, including assessment of the pupils, retina, and optic nerve, appears normal. Which of the following is the best method for predicting the visual outcome?

CT scan

Electroretinogram

Following the patient with sequential examinations

Visual evoked responses Please select an answer Feedback: An electroretinogram can aid in ruling out a retinal abnormality, but is unlikely to be helpful in this case since the damage is most likely at a higher level in the visual system. Many studies have attempted to use visual evoked response (VER) to help in determining visual prognosis in infants following cortical visual loss, but they have reported conflicting results. At the present time, it appears that the flash-evoked VER is not reliable enough to aid in predicting the visual outcome of infants who suffer a neurologic insult. CT scan or MRI of the brain may offer better clues than VERs in predicting visual outcome; however, the results are not conclusive. The best method for predicting a child's visual outcome appears to be sequential examinations during the first year or two of life.

Question 55 of 130 Tuberous sclerosis is not typically characterized by which of the following?

Epilepsy

Pseudosheathing of retinal vessels

Autosomal recessive inheritance

Skin lesions Please select an answer Feedback: The classic triad of tuberous sclerosis includes epilepsy, mental retardation, and characteristic skin lesions called adenoma sebaceum. Most evidence suggests that it is transmitted by an autosomal dominant gene with incomplete penetrance; it may also occur as a sporadic condition. Ocular involvement occurs in at least 50% of patients, with hamartomas of the retina and optic nerve as the primary manifestations. These lesions may be associated with pseudosheathing of the retinal vessels.

Question 56 of 130 Which of the following is not a classical feature of incontinentia pigmenti?

Increased incidence in male infants

Skin lesions presenting soon after birth

Peripheral retinal vascular anomalies

Retinal dysplasia Please select an answer Feedback: Incontinentia pigmenti is an uncommon familial disorder affecting the skin, bones, teeth, central nervous system, and eyes. It is thought to be inherited as an X-linked dominant disorder that is usually lethal in males, and therefore leads to a marked female predominance. Ocular abnormalities occur in about 35% of cases. Peripheral retinal vascular anomalies are common. The most serious complication is retinal dysplasia, which is associated with severe visual loss.

Question 57 of 130 Of the following case scenarios, which is the least likely to show visual improvement (≥ 20/40) with full-time occlusion therapy?

An 8-year-old child with a refraction of +3.50 OD and +1.00 OS whose best-corrected visual acuity is 20/200 OD and 20/25 OS

A 2-year-old child following surgery and contact lens fitting for a unilateral traumatic cataract of 4 months duration

A 4-year-old child with esotropia of unknown duration whose visual acuity is 20/25 OD and 20/200 OS

A 2-year-old child found on routine examination to have an anomalous optic nerve head OD, and who demonstrates poor fixation and following movements with that eye Please select an answer Feedback: The patients in the cases presented may have permanent visual impairment as a result of irreversible functional amblyopia, organic amblyopia, or a combination of both. A 4-year-old child with strabismic amblyopia should be very responsive to amblyopia therapy; more than 90% of cases achieve 20/40 visual acuity or better. Most studies also suggest that more than 80% of the time an older child with anisometropic amblyopia will also respond to aggressive occlusion therapy. A young child with a unilateral acquired cataract has a worse prognosis than either of the first two choices, but still may have a good result. In studies of acquired pediatric cataracts, between 54% and 79% of patients achieved ≥20/40 visual acuity. The worst prognosis appears to be a child with a unilateral optic nerve abnormality who presents with visual impairment. In one study of patients who presented with this clinical situation, the probability of obtaining a visual acuity of ≥20/40 was only 21%.

Question 58 of 130 A 2-year-old boy presents with a history of crossing of his left eye that has been noted by his parents over the past 3 months. Examination reveals a dense total cataract in the left eye. The anterior segment is otherwise normal. The right eye is normal. Both pupils react normally. The past medical history is unremarkable. What should be the next step?

Observation

Ocular ultrasound

Cataract extraction

Urinalysis for reducing substances Please select an answer Feedback: The most common causes of unilateral cataracts in the pediatric population are trauma, posterior lenticonus, and persistent hyperplastic primary vitreous. Other less common but important causes include intraocular inflammation (ie, toxocariasis) and intraocular tumor (ie, retinoblastoma). Urinalysis for non--reducing substances after ingesting milk is a screening test for ; galactosemia would be highly unlikely. Most children with galactosemia present in infancy with feeding intolerance and hepatosplenomegaly. By 2 years of age, they usually manifest moderate to severe mental retardation. It would also be unusual for a galactosemic cataract to present unilaterally. Cataract extraction most likely would be indicated in this case; however, prior to cataract extraction, it would be important to assess the posterior segment of the affected eye. Ocular ultrasound provides the best method for assessing the status of the vitreous and retina.

Question 59 of 130 A 30-year-old male accountant sustained a head injury as a result of an automobile accident 8 months previously. This patient's chief complaint is double vision when reading. There is no deviation in primary gaze, but a 15 prism diopter esotropia is present in down gaze. There is a left hypertropia of 8 prism diopters in right gaze, and a right hypertropia of 10 prism diopters in left gaze. Excyclotorsion of 6' is present in primary gaze, increasing to 15' in down gaze. Ocular versions demonstrate underaction of both superior oblique muscles. What is the best treatment option for this patient?

Reading glasses with prisms

Bilateral inferior oblique weakening procedures

Bilateral superior oblique tucks

Advancement of the anterior fibers of both superior oblique muscles Please select an answer Feedback: This patient demonstrates a common motility disturbance found with bilateral superior oblique palsies. These patients often fuse in primary gaze, but have difficulty with reading as a result of the esotropia and torsion in down gaze. Some patients can compensate satisfactorily by holding their reading material in primary position. When develops, however, bifocals do not work well, and full-frame reading glasses are necessary. In a young patient whose occupation requires considerable near work, this approach would not seem reasonable. Prisms would not be helpful in this case because of the torsion. A surgical procedure would offer the most effective method for correcting this patient's problem. Inferior oblique weakening procedures will have minimal effect on the deviation in down gaze. Advancement of the anterior fibers of the superior oblique muscle (Harada-Ito procedure) will correct torsion and possibly some of the esodeviation in down gaze, but will not significantly affect the vertical deviation on side gaze. Superior oblique tucks are the best procedure to improve the horizontal, vertical, and torsional misalignment problems.

Question 60 of 130 You are asked to evaluate in the newborn nursery a low-birth-weight infant with possible eye anomalies. During your examination, you instruct the nursing personnel to dilate the child's eyes with a mixture of 0.5% cyclopentolate and 2.5% phenylephrine ophthalmic solution. Which of the following additional instructions should not be given to the nursing personnel?

Temporarily occlude the nasolacrimal puncta after administering the drops

Delay the infant's feeding for at least 2 hours

Closely observe the vital signs

Administer or increase supplemental oxygen

Please select an answer

Feedback: A 2.5% solution of phenylephrine has been reported to result in a significant elevation of systolic blood pressure in premature and low-birth-weight infants. Similarly, cyclopentolate has been associated with paralytic ileus and, at least in one instance, death. Based on this knowledge, it is reasonable to observe infants closely when these ocular drugs are being administered. Occlusion of the puncta will help decrease systemic absorption. Delaying feeding before administering the drops has been shown to reduce the incidence of gastrointestinal problems. Respiratory distress is not a common side effect of either of these drugs, and the administration of supplemental oxygen does not appear to be indicated unless the child demonstrates a decrease in PaO2 during the examination. A commercially available solution of 0.2% cyclopentolate and 1.0% phenylephrine seems to provide adequate in neonates with fewer side effects than the higher concentrations

Question 61 of 130 Well recognized clinical features of Brown syndrome include which of the following findings on examination?

Deficient elevation in adduction that worsens with abduction

Deficient elevation in abduction that improves with adduction

Deficient elevation in abduction that worsens with adduction

Deficient elevation in adduction that improves with abduction Please select an answer Feedback: Well recognized clinical features of Brown syndrome include deficient elevation in adduction that improves with abduction, which is an important point of difference from inferior oblique palsy. Positive forced duction testing is also found in individuals with Brown syndrome, which further distinguishes the condition from inferior oblique muscle palsy. In adduction, a downshoot of the involved eye and widening of the palpebral fissure is commonly seen, which distinguishes the condition from superior oblique muscle overaction, because downshoot with superior oblique overaction occurs less abruptly as adduction is increased.

Question 62 of 130 A 9-year old boy presents for further evaluation of leukocoria (white pupil) in the right eye, which was first noted by the child’s parents in a recent photograph. The child is otherwise healthy and his past medical history is unremarkable. Examination reveals the presence of abnormal retinal vessels with yellow subretinal exudates in the child’s right eye. The patient’s left eye appears to be normal on examination. Which of the following is the most likely cause of the patient’s condition?

Organized vitreous hemorrhage

Retinoblastoma

Coats disease

Retinopathy of prematurity Please select an answer Feedback: Coats disease can mimic retinoblastoma, but is understood to mean the presence of abnormal retinal vessels with a fundus picture showing yellow subretinal exudates. In the juvenile form of Coats disease, males tend to be affected more frequently than females. The condition is commonly, but not always, unilateral. The typical age of onset of juvenile Coats disease is 8-10 years; however, the disease has also been described in infants. The diagnosis of Coats disease requires the presence of abnormal retinal vessels, which can be small and difficult to find without fluorescein angioscopy and angiography. Treatment is directed at obliterating the abnormal vessels.

Question 63 of 130 A 13-year old female presents with asthenopia, blurred near vision, and difficulty reading. On examination, the patient is noted to have an exophoria at near, a remote near point of convergence, and poor near fusional convergence amplitudes. Which of the following is the most appropriate next step in treatment?

Plus lenses and base-in prisms for reading

Orthoptic exercises (“pencil push-ups”)

Bilateral lateral rectus muscle recessions

Bilateral medial rectus muscle resections Please select an answer Feedback: Treatment of convergence insufficiency often includes orthoptic exercises, particularly near point exercises such as “pencil push-ups” or stereograms. Base-in prism reading glasses are usually used if the patient fails to respond to orthoptic exercises. When patients have combined convergence and accommodative insufficiency, plus lenses and base-in prisms for reading may be beneficial. Medial rectus muscle resections are used rarely when nonsurgical treatments have been unsatisfactory. Lateral rectus muscle recessions are not usually used for treatment of convergence insufficiency.

Question 64 of 130 An infant girl presents for evaluation with epiphora, blepharospasm, and photosensitivity. On examination, the infant is noted to have corneal clouding in the right eye, and the right appears to be enlarged. Which of the following is the most likely cause of this patient’s condition?

Primary congenital glaucoma

Conjunctivitis

Corneal abrasion

Nasolacrimal duct obstruction Please select an answer Feedback: The “clinical triad” of primary congenital glaucoma includes epiphora, , and blepharospasm. Corneal clouding and enlargement of the cornea are other common findings. Corneal clouding is secondary to edema resulting from elevated intraocular pressure, and the presence of corneal clouding may be gradual or sudden in onset. Photophobia, epiphora, and blepharospasm result from the glare and epithelial abnormalities associated with corneal edema and opacification. Corneal enlargement results form gradual stretching of the cornea as a result of elevated intraocular pressure. Corneal enlargement tends to occur in older infants up to about 2-3 years of age. A corneal diameter of greater than 11.5 mm is suggestive of glaucoma in a newborn, and glaucoma should be suspected in any child with a corneal diameter greater than 13 mm.

Question 65 of 130 A 12-year old male presents with asthenopia, blurred near vision, and difficulty reading. On examination, the patient is noted to have an exophoria at near. The patient also has a remote near point of convergence, and poor near fusional convergence amplitudes. Which of the following is the most likely diagnosis?

Accommodative spasms

Convergence insufficiency

Convergence paralysis

Intermittent exotropia Please select an answer Feedback: Characteristics of convergence insufficiency include asthenopia, blurred near vision, and reading problems. The condition typically occurs in older children and adults. Findings on examination in patients with convergence insufficiency include poor near fusional convergence amplitudes and a remote near point of convergence. Patients may also have an exophoria on examination, but should not have an exotropia. Convergence paralysis is usually secondary to an intracranial lesion and is a condition distinct from convergence insufficiency. Accommodative spasm refers to the inability to suitably relax the ciliary muscle resulting in blurred vision, asthenopia, headache, or seeming increase in myopia.

Question 66 of 130 A 3-year old boy presents for evaluation with a three week history of intermittent esotropia. On sensorimotor exam there is 20 prism diopters of esotropia at distance and near. Cycloplegic retinoscopy reveals +4.00 D hyperopia in each eye. Which of the following is the most appropriate next step in treatment?

Partial correction of the hyperopia

Correction of the full amount of hyperopia

Eyeglasses combined with surgical correction

Surgical correction of the esotropia Please select an answer Feedback: The patient described above has accommodative esotropia due to significant refractive error, also known as refractive accommodative esotropia. The angle of esotropia tends to be approximately equal at distance and near, and the amount of hyperopia averages +4.00 D, but can range from +3.00 D to +10.00 D. Treatment of refractive accommodative esotropia consists of correction of the full amount of hyperopia determined under cycloplegia with cyclopentolate or atropine. If patients with accommodative esotropia are found to have amblyopia, amblyopia therapy should also be initiated.

Question 67 of 130 A 10-month old, full-term gestation, and previously healthy child, presents for further evaluation of leukocoria (white pupil) in his left eye, which was first noted by the child’s parents in a recent photograph. A CT scan shows evidence of a mass with localized calcification in the left eye. Which of the following is the most likely cause of the patient’s condition?

Organized vitreous hemorrhage

Retinopathy of prematurity

Coats disease

Retinoblastoma Please select an answer Feedback: Retinoblastoma is the most common malignant ocular tumor of childhood and one of the most common of all pediatric solid tumors. In familial and bilateral cases, retinoblastoma tends to be diagnosed in the first year of life, whereas retinoblastoma tends to be diagnosed between 1 and 3 years of age in sporadic unilateral cases. Leukocoria (white pupil) is the most common initial sign, and about 25% of cases present with strabismus as the first sign. Less common presentations include vitreous hemorrhage, , glaucoma, proptosis, ocular inflammation, and hypopyon. The demonstration of intraocular calcification by CT scan usually confirms the diagnosis.

Question 68 of 130 A 2-year old child is found to have a painless mass located superotemporal to the globe, which is unattached to the overlying skin. The mass is mobile, smooth, and nontender. The child is diagnosed with a dermoid cyst, and the diagnosis is supported on CT scan, which shows a well-circumscribed lesion with a low-density lumen. Which of the following is the most appropriate next step in treatment?

Systemic corticosteroid

Surgical excision

Observation only

Topical corticosteroid Please select an answer Feedback: Dermoid cysts are the most common space-occupying lesions of childhood and are benign developmental choristomas. The cysts are congenital, arising from rests of primitive dermal elements that have been sequestered in fetal suture lines at the time of closure. The cysts are lined with keratinized epithelium and dermal appendages including hair follicles, sweat glands, and sebaceous glands. Dermoid cysts present as a painless mass that is unattached to overlying skin and is mobile, smooth, and nontender. CT scan can confirm the diagnosis. Treatment of dermoid cysts is surgical; however, surgery may be delayed until the risk of general anesthesia is outweighed by the possibility of accidental rupture of the cyst from trauma.

Question 69 of 130 A 4-year old girl presents for evaluation of esotropia. The child is found to have 20/25 vision at distance and near. Corneal light reflex tests and cover testing results are normal. Which of the following is the most likely diagnosis?

Intermittent esotropia

Pseudoesotropia

Intermittent exophoria

Malingering Please select an answer Feedback: Patients with pseudostrabismus appear to have esotropia or exotropia when in fact their eyes are straight. Both corneal light reflex testing and cover testing results are normal, because no true ocular misalignment exists. Infants tend to have wide, flat nasal bridges with prominent epicanthal folds and small interpupillary distances, which can give the false appearance of crossed eyes. In most cases, the appearance of ocular misalignment gradually improves during infancy and childhood. It is important to note that children with pseudostrabismus can also have intermittent ocular misalignment. Therefore, the report from a parent that the child’s eyes do not appear straight should not be taken lightly

Question 70 of 130

Superior Oblique muscle palsy (fourth or trochlear nerve palsy) is the most common isolated cyclovertical muscle palsy encountered by ophthalmologists. Which of the following conditions is the most common cause of acquired superior oblique muscle palsy?

Direct trauma to the tendon

Closed head trauma

Cavernous sinus aneurysm

Longstanding hypertension

Please select an answer

Feedback: Superior oblique palsy may be congenital or acquired. Congenital superior oblique palsy often results from a defect in the fourth nerve nucleus, or a defect in the motor portion of the fourth cranial nerve. Acquired superior oblique palsy most commonly occurs following closed head trauma. Rarely, acquired superior oblique palsy can result from central nervous system vascular problems, , or brain tumors. Direct trauma to the tendon or trochlea is an unusual cause of superior oblique palsy.

Question 71 of 130 An 8-year-old boy presents for evaluation with a 3-day history of fever, lethargy, headache, eye pain, and eyelid edema. The child’s past medical history is significant for sinusitis. On examination, the child appears uncomfortable. He is found to have 20/20 vision with his right eye and 20/40 vision with his left eye. Although the patient has full ocular motility with his right eye, there is proptosis and limitation of ocular motility with his left eye. The child also expresses pain in his left eye on ocular motility examination. Which of the following is the most appropriate initial step in treatment?

Close monitoring with referral to a pediatric otolaryngologist

Daily monitoring in the office and treatment with oral antibiotics

Urgent referral to radiology for orbital computed tomography

Hospitalization and treatment with intravenous antibiotics Please select an answer Feedback: The patient described above has a left . Orbital cellulitis is commonly associated with ethmoid or frontal sinusitis. Early signs and symptoms of orbital cellulitis include fever, lethargy, eyelid edema, rhinorrhea, headache, orbital pain, and tenderness on palpation. Unlike preseptal cellulitis, patients with orbital cellulitis also have proptosis and limited ocular motility on exam. Orbital cellulitis in children is a serious disease requiring hospitalization and treatment with intravenous antibiotics. Orbital computed tomography is important to perform, but it is not the most appropriate initial step in treatment. It is reasonable to consult an otolaryngologist due to the patient’s history of sinusitis, especially if sinus disease is determined on computed tomography. However, the process of consulting an otolaryngologist should not delay the patient’s hospitalization and treatment with intravenous antibiotics. Treatment of orbital cellulitis in a child in an outpatient setting is not appropriate.

Question 72 of 130 Which one of the following is expected with stereopsis testing in a 12-year-old girl diagnosed with well-controlled intermittent exotropia?

Decreased stereopsis with episodes of diplopia

Excellent stereopsis

Decreased stereopsis with hemiretinal suppression

Monofixation syndrome Please select an answer Feedback: Excellent stereopsis is expected to exist in the presence of well-controlled, intermittent exotropia. When intermittent exotropia is well controlled, the deviation manifests itself only after disruption of fusion, such as after cover testing. Once the deviation becomes manifest, the patient often can resume fusion rapidly. In the presence of well-controlled intermittent exotropia, decreased stereopsis is not typically found on clinical exam. Monofixation syndrome occurs in individuals with a macular scotoma in one eye, good peripheral fusion, and gross stereopsis. The characteristics of monofixation syndrome are not seen in patients with well-controlled intermittent exotropia.

Question 73 of 130 A 21-year-old woman presents with exotropic Duane's syndrome. Which one of the following surgical options is most likely to benefit this patient?

Recession of the lateral rectus muscle in the involved eye

Resection of the medial rectus muscle in the involved eye

Recession of the lateral rectus muscle in the uninvolved eye

Resection of the medial rectus muscle in the uninvolved eye Please select an answer Feedback: Recession of the lateral rectus muscle in the involved eye is most likely to benefit patients with exotropic Duane's syndrome in the presence of primary position deviation. The procedure also eliminates any existing head turn. When patients with exotropic Duane's syndrome have a deviation greater than 20 prism diopters in primary position, recession of the lateral rectus muscle in both eyes should be performed for maximum benefit. Resection of the medial rectus muscle in the involved eye will worsen globe retraction in patients with exotropic Duane’s syndrome, and for this reason, the procedure should be avoided. Recession of the lateral rectus muscle in the uninvolved eye alone will have minimal to no effect on the patient’s ocular alignment. Resection of the medial rectus muscle in the uninvolved eye will also have minimal to no effect.

Question 74 of 130 A 3-month-old infant presents with a history of constant epiphora since birth. On examination the patient appears relatively comfortable and is found to have epiphora with mucopurulent discharge from both eyes. Fluorescein solution is placed into the tear film of each eye, and the patient is noted to have significant retention of the fluorescein dye with failure of the dye to appear in the nose after 15 minutes. The remainder of the exam is normal. Which of the following is the most likely diagnosis?

Bilateral conjunctivitis

Congenital nasolacrimal duct obstruction

Congenital glaucoma

Bilateral punctual or canalicular atresia Please select an answer Feedback: Obstruction of drainage below the lacrimal sac tends to occur in 5% of newborns and is usually caused by a thin mucosal membrane at the distal end of the nasolacrimal duct. Symptoms consist of epiphora and mucoid or mucopurulent discharge. The impatency of the drainage system can be confirmed by placing fluorescein solution in the tear film and noting retention of the dye in the tear film after 5-10 minutes. The dye will also fail to appear in the nose or pharynx after 10-15 minutes. Conservative management involves nasolacrimal massage with topical broad spectrum antibiotics. Initial surgical management involves nasolacrimal duct probing. Conjunctivitis may have discharge that is serous, mucopurulent, or purulent. Patients with conjunctivitis tend to have burning, stinging, foreign body sensation, ocular discharge, and matting of the eyelids. Congenital glaucoma is associated with epiphora, photophobia, and blepharospasm. The cornea also tends to be enlarged with corneal clouding. Punctal atresia is usually ruled out by inspection.

Question 75 of 130 A 16-year-old girl presents with acute, bilateral eye pain, redness, and globe prominence, right > left, with mild of the right upper eyelid. Both eyes have restricted motility in all fields of gaze. Blood testing reveals normal T3, T4, TSH, WBC, CRP, and ANA. Which of the following orbital CT findings would be most consistent with her presentation and lab results?

Thickening of the posterior and all recti including the tendons

Ethmoid sinusitis with medial orbital abscesses, right > left

Fusiform enlargement of the optic nerve sheaths, right > left

Thickening of the extraocular muscles: inferior > medial rectus muscles, right > left Please select an answer Feedback: results in granulomatous inflammation of the sclera. Scleritis is not common in children. When present, scleritis is characterized by deep eye pain, pain with eye movement, ocular erythema, and mild proptosis. Serious retinal detachments may occur resulting in decreased vision. Scleritis is classified as either anterior scleritis or posterior scleritis. Anterior scleritis is further classified as non-necrotizing (which may be diffuse or nodular) or necrotizing (with or without inflammation).

Question 76 of 130 A 13-month-old girl is brought to the emergency room for evaluation of seizures and difficulty breathing after falling from a bed at home. The emergency room physician suspects shaken baby syndrome. Which one of the following is considered the most common ocular manifestation of shaking injury?

Full-thickness retinal detachment

Retinal hemorrhage

Lens dislocation with hyphema

Periocular bruising Please select an answer Feedback: Victims of shaking injury are almost always younger than 3 years, and shaken baby syndrome is recognized as one of the most important manifestations of child abuse. Violent shaking of children younger than 3 years results in ocular, intracranial, and sometimes physical injuries in the abused child. The most common ocular manifestation, in the majority of cases, is retinal hemorrhage. Full-thickness retinal detachment and full-thickness retinal breaks are rare features of shaken baby syndrome. External evidence of trauma, such as periocular bruising, is typically lacking, and the anterior segment tends to appear normal.

Question 77 of 130 A 7-year-old boy presents for evaluation with a 1-week history of progressive proptosis of his right eye. In addition, he is noted to have ptosis of his right upper eyelid, erythema of the right upper eyelid, and right eyelid edema. On sensorimotor exam there is 15 prism diopters of exotropia at distance and near. There is no history of fever, lethargy, or headache. Which one of the following is the most likely diagnosis?

Retinoblastoma

Rhabdomyosarcoma

Orbital cellulitis

Metastatic neuroblastoma Please select an answer Feedback: Rhabdomyosarcoma is the most common pediatric orbital malignancy, exceeding all other sarcomas combined. The average age of onset is 5-7 years. The presenting sign is usually proptosis, which can develop over a few days. Strabismus and ptosis are also common. Biopsy is required for confirmation of the diagnosis when rhabdomyosarcoma is suspected, and CT or MRI should be obtained to help delineate the tumor to plan the best approach for biopsy. Early diagnosis and treatment of this tumor can be life-saving. Orbital cellulitis is commonly associated with ethmoid or frontal sinusitis. Early signs and symptoms of orbital cellulitis include fever, lethargy, eyelid edema, rhinorrhea, headache, orbital pain, and tenderness on palpation. When metastatic neuroblastoma presents, the clinical features vary according to the site or origin of the tumor. Pain, fever, and weight loss are common symptoms. Unilateral or bilateral proptosis and lid ecchymosis are classic presentations of metastatic neuroblastoma. The mean age at diagnosis of patients with neuroblastoma is 2 years. Retinoblastoma is the most common malignant ocular tumor of childhood. It is typically diagnosed during the first year of life in familial and bilateral cases, and between 1 and 3 years of age in sporadic cases. Onset later than 5 years of age is rare. The most common initial sign is leukocoria. Less commonly, patients present with vitreous hemorrhage, hyphema, ocular or periocular inflammation, glaucoma, proptosis, and hypopyon.

Question 78 of 130 A 2-year old child is at greatest risk for developing anisometropic amblyopia with which of the following uncorrected refractive errors?

OD = +1.00 +1.00 x 90; OS = +2.75 +1.00 x 090

OD = -1.00 +1.00 x 90; OS = -2.75 –1.00 x 180

OD = +1.25 +1.25 x 90; OS = +2.75 –1.00 x 180

OD = -1.00 +1.00 x 90; OS = -2.75 +1.00 x 090 Please select an answer Feedback: Minimal degrees of myopia seldom cause amblyopia, as there is good visual development at near. The larger degree of anisohyperopia puts the patient for greatest risk of developing amblyopia.

Question 79 of 130 A 4-year-old girl is found to have distance visual acuity of 20/50 in each eye. Cycloplegic retinoscopy reveals +6.50 hyperopia in her right eye and +6.00 hyperopia in her left. She is also found to have 10 prism diopters of esophoria at distance and near. The remainder of the eye exam is unremarkable. Which one of the following is the most likely cause of the patient’s decreased vision?

Amblyopia

Strabismus

Anisometropia

Malingering Please select an answer Feedback: Amblyopia is caused by abnormal visual experience early in life and can affect one or both eyes. Amblyopia can result from visual deprivation, anisometropia, high bilateral refractive errors (isometropia), or strabismus. When amblyopia is bilateral it results from large, approximately equal, uncorrected refractive errors in both eyes of a young child. Hyperopia greater than 5 diopters and myopia greater than 10 diopters increase a child’s risk for bilateral amblyopia. Treatment requires correcting the child’s refractive error. Although this patient does not have significant anisometropia, anisometropia of 1-2 diopters can induce mild amblyopia. Strabismus results in amblyopia in the consistently deviating eye of a child such as esotropia. Given the results of the cycloplegic retinoscopy, it is unlikely the patient is malingering.

Question 80 of 130

A 6-month-old girl presents for evaluation of esotropia, which has been present since birth. Examination reveals 40 prism diopters of esotropia. The patient is also noted to cross-fixate. Which one of the following is the most appropriate initial therapy?

Alternate occlusion for treatment of amblyopia

Surgical correction before 1 year of age

Surgical correction between 2 and 5 years of age

Spectacles for correction of hyperopia

Please select an answer

Feedback: Early surgery for infantile esotropia is believed to maximize binocular visual function. Gerth et al. determined that early surgery at or before 11 months of age promotes the development of cortical visual motion processing compared with standard surgery at age 11 to 18 months. The most commonly performed procedure for infantile esotropia is bilateral medial rectus muscle recession. Cycloplegic refraction tends to reveal 1-2 diopters of hyperopia, which is considered normal refractive error in young children. Thus, spectacles for correction of hyperopia is not the most approriate initial therapy for patients with infantile esotropia. Although occlusion therapy is used to treat amblyopia, alternate occlusion therapy is not the most appropriate initial therapy for either infantile esotropia or amblyopia.

Question 81 of 130

A 3-year-old girl has an esotropia of 20-prism diopters (PD) in primary position, 15 PD in upgaze, and 30 PD in downgaze. In addition to operating on the medial rectus muscle in each eye, which of the following is an appropriate treatment for her V-pattern?

Weaken both inferior oblique muscles

Weaken both superior oblique muscles

Infraplace both lateral rectus muscles

Supraplace both medial rectus muscles Please select an answer Feedback: Correction of A or V patterns depends on the amount of over- or underaction of the oblique muscles, which can be assessed, in part by testing the ocular versions. Small to moderate A and V patterns are characterized by normal oblique muscle function and are corrected by vertically displacing the horizontal rectus muscles. The medial recuts muscles are displaced downward for V-pattern correction and the lateral recuts muscles are displaced upward. Large V patterns (a difference of 15 prism diopters or more between up and down gaze) associated with oblique muscle dysfunction usually require oblique muscle surgery. The use of superior oblique weakening procedures is inappropriate for treatment of V-patterns.

Question 82 of 130 Despite maximal medical management and prior surgery, a 5-year-old boy with Sturge-Weber syndrome presents for evaluation of progressive glaucoma OD. Anterior segment examination shows the are clear with horizontal diameters 13 mm OD, 11 mm OS. Intraocular pressures measured 26 mmHg OD, 17 mmHg OS. Which treatment should be considered next?

Goniotomy

Drainage implant

Trabeculotomy

Trabeculectomy Please select an answer Feedback:

Glaucoma is the most common and serious complication in children with Sturge-Weber syndrome. Glaucoma develops in about half the cases. Therapy with topical drops can be effective, but when medical management fails, surgery is indicated. Setons or aqueous drainage devices have been shown to be useful for management of intractable glaucoma in patients with Surge-Weber syndrome. Goniotomy and trabeculotomy are not appropriate treatment methods in patients with Sturge-Weber syndrome who have failed previous surgical and medical management. Trabeculectomy in the absence of mitomycin-C is also unlikely to be successful in this patient.

Question 83 of 130 A 4-1/2-year-old girl presents with a of 3 weeks duration. The vision is light-perception, there is no view of the posterior pole, and the other eye is normal. Given the examination shown, which of the investigations and interventions should not be indicated?

An anterior chamber tap for cell identification and histopathology

Angiotensin-converting enzyme (ACE) titer

Systemic evaluation for arthritis

CT scan of the head and orbits Please select an answer Feedback: The pseudohypopyon in the figure is composed of retinoblastoma cells. The lack of view of the posterior pole is a contraindication to an anterior chamber tap without further investigations, such as CT scan or ocular ultrasound to rule out the presence of a posterior-pole tumor. Surgical entry into an eye harboring a retinoblastoma can result in systemic metastasis. This child is slightly older than might be expected for a unilateral retinoblastoma; 90% present by age 3 years. There are cases of unilateral retinoblastoma, however, that initially present in late childhood. Retinoblastoma can present as a -like condition and even as a fulminant orbital cellulitis. A workup for juvenile rheumatoid arthritis and sarcoidosis is reasonable in the presence of childhood uveitis, but this particular child has a pseudohypopyon of large, white tumor cells that are atypical of bilateral granulomatous uveitis. Leukemia can also present as a hypopyon and must be included in the differential diagnosis for this patient. However, in this case, a CT scan of the child's orbits demonstrated a large calcified intraocular mass consistent with retinoblastoma.

Question 84 of 130 An otherwise healthy 6-month-old child is diagnosed with a 2-mm epibulbar dermoid tumor of his left eye. The child is playful and appears asymptomatic. Which of the following is the most appropriate next step in the management of this patient?

Ultrasound biomicroscopy

Immediate surgical excision

Observation

Topical lubrication Please select an answer Feedback: There is no urgency in removing epibulbar dermoids unless symptoms or management of localized inflammation are present, and persist despite medical treatment. The most frequent symptom is local irritation, which can be managed with topical lubricants. Significant astigmatism can also result which can require spectacle correction. Contact lens wear may become limited due to irritation of the lens rubbing on the dermoid. Surgical removal of epibulbar tumors can result in residual corneal scarring.

Question 85 of 130

A 1-year-old boy with a prior unilateral catarct in his left eye presents for evaluation. His cataract was diagnosed at birth and he underwent lensectomy. Exam findings are consistent with severely decreased vision in his left eye. What is the most likely cause?

Ametropic amblyopia

Anisometropia amblyopia

Deprivation amblyopia

Strabismic amblyopia Please select an answer Feedback: Deprivation amblyopia is caused by any abnormality that occludes the visual axis and prevents the formation of a clear, focused retinal image during the critical period of visual development. Examples include visually significant cataracts, corneal opacities, ptosis and other lid disorders, and excessive amblyopia treatment. Anisometropic and ametropic amblyopia are forms of refractive amblyopia. Anisometropic amblyopia results from unequal refractive error between the two eyes. Ametropic amblyopia results in bilateral reduction in vision from large, approximatley equal, uncorrected refractive errors in both eyes of young children. Strabismic amblyopia results from ocular misalignment.

Question 86 of 130 A 2-week-old girl is referred for evaluation of an abnormal red reflex OS. Examination reveals a dense, posterior, 6- mm central lenticular opacity OS with normal retinal examination. Cycloplegic retinoscopy reveals +2.00 sphere hyperopia in each eye. What step should be considered next?

Spectacles

Lensectomy with anterior vitrectomy

Daily pharmacologic dialation

Observation Please select an answer Feedback:

Cataracts present at birth are serious because the visual system is not fully developed, putting the patient at risk for irreversible amblyopia if the visual axis is not cleared within 6 to 8 weeks of birth. Unilateral cataracts are more likely than bilateral opacities to cause permanent visual loss. Daily pharmacologic dilation is an inappropriate treatment for patients with visually significant cataracts (3 mm or greater). Spectacles and observation are also inappropriate initial management options for this patient.

Question 87 of 130 A 6-week-old infant presents with a bluish mass in the medial canthus that has been present since birth (see the figure). A pediatric otorhinolaryngologist evaluated the patient and noted a mass lesion in the nose. What is the most likely diagnosis?

Amniotocele

Metastatic neuroblastoma

Nasal encephalocele

Dermoid Please select an answer Feedback: The location and appearance of this bluish mass in the region of the lacrimal sac is a classical presentation for an amniotocele. This disorder represents a congenital proximal and distal obstruction of the lacrimal system that can become infected. Because of the obstructions this can lead to an absces, followed by . An amniotocele can appear as a mass in the nose and can cause respiratory symptoms. Nasal encephaloceles are rare, but should be included in the differential diagnosis. However, the location of lacrimal sac amniocele is the most common locatiton to present in this manner. The location and appearance also make dermoid or neuroblastoma very unlikely; clinical examination should help distinguish these entities. Gentle massage of an amniotocele may result in expression of material from the nasolacrimal duct which can decompress the mass. In some cases, a CT scan or MRI may be indicated. Although some cases have resolved with only massage treatment, the presence of an amniotocele is usually an indication for nasolacrimal duct probing.

Question 88 of 130 In the nonverbal patient, what method or test reveals amblyopia?

Forced preferential looking

Electroretinogram (ERG)

Bagolini lenses

A 4-prism diopter base-out test

Please select an answer Feedback: Forcded preferential looking can assess the presence, laterality, or absence of amblyiopia. It is most sensitive to detect unilateral amblyopia. Bagolini glasses assess rotatory diplopia. The 4-prism base out test is useful to detect microstrabismus (but not amblyopia). The ERG is useful to objectively assess retinal function and may be useful to further evaluate patients with fixation difficulty in each eye.

Question 89 of 130 A 10-year-old child is diagnosed with bilateral . What clinical feature is most likely to develop in this patient?

No change in visual acuity

Peripheral visual field loss

Elevated intraocular pressure

Progressive myopia Please select an answer Feedback: Children with papilledema require a full evaluation, including neuroimaging and lumbar puncture. Papilledema in children may be caused by hydrocephalus, an intracranial mass, or pseudotumor cerebri. Disease- related complications include loss of visual field and loss of visual acuity. Progressive myopia and elevated intraocular pressure are not likely to develop secondary to papilledema.

Question 90 of 130

A 4-year-old child presents for evaluation of a recent onset esotropia. What step should be considered next?

Cycloplegic refraction

Dry retinoscopy

Manifest refraction with simultaneous "pushing plus"

Autorefraction with the use of accommodative control

Please select an answer

Feedback: Cycloplegic refraction is used to determine the objective determination of refractive error by relaxing accommodation. It is especially important in young children and patients with strabismus. Dry retinoscopy, manifest refraction, and autorefraction are not satisfactory techniques to fully assess refractive errors in young children or patients with strabismus.

Question 91 of 130 An 8-year-old boy presents after a weekend of fever, lethargy, headache, eye pain, and eyelid edema. The child has had prior sinusitis. He is 20/20 in his right eye and 20/40 in his left eye. He has proptosis and limitation of ocular motility with his left eye. When attempting to move his left eye he complains of pain. What is the preferred first-line treatment?

Daily monitoring in the office and treatment with oral antibiotics

Urgent referral to radiology for orbital computed tomography

Close monitoring with referral to a pediatric otolaryngologist

Hospitalization and treatment with intravenous antibiotics Please select an answer Feedback: The patient has a left orbital cellulitis. Orbital cellulitis is commonly associated with ethmoid or frontal sinusitis. Early signs and symptoms of orbital cellulitis include fever, lethargy, eyelid edema, rhinorrhea, headache, orbital pain, and tenderness on palpation. Unlike preseptal cellulitis, patients with orbital cellulitis also have proptosis and limited ocular motility on exam. Orbital cellulitis in children is a serious disease requiring hospitalization and treatment with intravenous antibiotics. Orbital computed tomography is important to perform, but it is not the most appropriate initial step in treatment. It is reasonable to consult an otolaryngologist due to the patient’s history of sinusitis, especially if sinus disease is determined on computed tomography, however, the process of consulting an otolaryngologist should not delay hospitalization and treatment with intravenous antibiotics. Treatment of orbital cellulitis in a child in an outpatient setting is inappropriate.

Question 92 of 130

A 5-year-old girl with hydrocephalus is diagnosed with bilateral optic atrophy. What changes in her vision and findings are most likely to develop?

Initial optic nerve pallor with improvement over time

Progressive loss of all vision

Initial visual field defects with improvement over time

Progressive improvement of vision Please select an answer Feedback:

In children, optic atrophy usually results from disease of the anterior visual pathway. Workup should include neuroimaging for cases where the cause of optic atrophy is uncertain. A tumor or hydrocephalus is present in more than 40% of cases in this setting. Children with hydrocephelus-associated optic atrophy experience progressive loss of vision and visual field defects unless successfully shunted. Children with optic atrophy have a pale but normal sized optic nerve head.

Question 93 of 130 A 9-year-old boy with sickle cell trait is diagnosed with a 5-mm hyphema OD. He is coooperative during the examination, and intraocular pressures are 39 mm Hg OD and 12 mm Hg OS. What is the preferred first-line treatment?

Daily monitoring with sedatives

Eye examination under anesthesia

Pressure-lowering medications

Daily monitoring with topical corticosteroids Please select an answer Feedback: Patients with sickle cell trait or disease may develop sickling in the anterior chamber, resulting in elevated intraocular pressure, or retinal circulation, resulting in vascular occlusion. Pressure-lowering medication is appropriate for eyes with elevated intraocular pressure. The use of topical cotrticosteroids is currently unproved, but can be used to reduce the risk of inflammatory complications. Limitation of activity (especially normal childhood running, jumping, and rough play) is appropriate. An eye examination under anesthesia is not necessary in a cooperative patient. Sedatives are not usually used as a first line of treatment.

Question 94 of 130 A 12-year-old boy undergoes bilateral medial rectus muscle recessions for esotropia. At his postoperative visit, he is found to have widening of the palpebral fissure and an adduction deficit in the left eye. Ductions and versions are otherwise normal. He also is found to have 45 prism diopters of exotropia in primary position. What would be the preferred management?

Observation with plans for exploration in 1 to 2 weeks if not improved

Bilateral lateral rectus muscle recession to treat the exotropia

Prompt surgical exploration of the patient's left eye

Surgical exploration of both eyes following observation Please select an answer Feedback: Failure to adequately secure the muscles during strabismus surgery can cause the muscle to slip posteriorly within its capsule. This results in the patient being unable to move the eye in the direction of action of the affected muscle. Surgery on the affected eye is required as soon as possible to replace the slipped muscle. Prompt surgical treatment will restore the patient’s ocular motility and improve ocular alignment. As a result, there is no need for surgery on the lateral rectus muscles in the presence of a slipped medial rectus muscle. Observation would be inappropriate because it would allow the slipped muscle to further retract.

Question 95 of 130 Under anesthesia, a 6-month-old boy is found to have corneal diameters of 13 mm OD and 12.5 mm OS. Significant bilateral corneal edema is also present. Intraocular pressures (IOPs) are 33 mmHg OD and 29 mmHg OS. What would be the preferred first-line treatment?

Cycloablation

Goniotomy

Trabeculotomy

Trabeculectomy with mitomycin-c Please select an answer Feedback: Sugical intervention is the treatment of choice fort primary congenital glaucoma. Trabeculotomy is preferable in cases of poor visualization of the angle secondary to corneal edema. Goniotomy involves incising the trabecular meshwork under direct visualization and is often preferred when the cornea is clear. Trabeculectomy with the use of mitomycin-C is often not successful in patients younger than one year. Cycloablation is reserved for extremely resistrent cases that are not responsive to other treatment modalities.

Question 96 of 130 A 10-year-old child has open angle glaucoma and asthma. Which of the following adrenergic agents is most appropriate for managing this patient’s glaucoma?

Brimonidine

Timolol

Apraclonidine

Betaxlol Please select an answer Feedback: Brimonidine is an alpha-2 adrenergic antagonist. It effectively reduces intraocular pressure in some cases of pediatric glaucoma, but can cause lethargy, , hypothermia, and serious CNS depression in young children. Iopidine is an alpha-2 adrenergic agonist that has been useful for short-term intraocular pressure reduction in children, but tends to result in tachyphylaxis and allergy in young children. Timolol and betaxolol are topical beta- blockers and should not be used in children with asthma.

Question 97 of 130

A 15-year-old boy has bilateral high myopia, a sensory exotropia, and a history of retinal detachment in the right eye requiring scleral buckling. He recently had a recession/resection muscle strabismus surgery in the right eye for his exotropia. On postoperative exam, his right eye has corneal edema and folds in Descemet’s membrane, and anterior inflammation. What is the most likely diagnosis?

Normal postoperative inflammation

Anterior segment ischemia

Keratitis

Traumatic anterior uveitis Please select an answer Feedback:

Anterior segment ischemia is characterized by corneal epithelial edema, folds in Descemet’s membrane, and additional signs of anterior uveitis. The risk for anterior segment ischemia increases following simultaneous surgery on three or more rectus muscles in the same eye or two rectus muscles in the same eye in patients with poor blood flow. Treatment involves the use of topical, subconjunctival, or systemic steroids. Normal postoperative inflammation does not result in corneal epithelial edema, folds in Descemet’s membrane, or significant anterior uveitis. Keratitis is not seen following uncomplicated strabismus surgery.

Question 98 of 130 A 6-month-old infant boy is diagnosed with congenital esotropia. For what complication is the boy at greatest risk?

Abduction deficit

High hyperopia

Amblyopia

Myopia Please select an answer Feedback: Esotropia in a 6-month-old is most consistent with a diagnosis of infantile, or congenital, esotropia. Patients with infantile esotropia tend to have relatively large deviation (30 prism diopters or greater) and do not have abduction deficits. Abduction can be difficult to test in an infant with this condition and may require stimulation of the oculovestibular reflex or a trial with monocular occlusion. Accommodative esotropia has been reported in children as young as 4 months of age and can usually be distinguished by a trial with glasses if a refractive error of more than 2 diopters of hyperopia is found. Myopia is not a common finding in infants with congital esotropia. Amblyopia occurs in approximately 40% of children with infantile esotropia both before and after surgical correction. Although surgery is the treatment of choice for infantile esotropia, a high percentage of patients develop a postsurgical accommodative component over time. In one study, 65% of surgically corrected patients with congenital esotropia required spectacle correction of their hyperopia to control the ocular alignment.

Question 99 of 130 An infant is diagnosed with optic nerve hypoplasia. What is the first evaluation to be ordered?

Photographs of the optic nerves

Genetic testing

CT Scan

Endocrine consultation Please select an answer Feedback: Optic nerve hypoplasia can be unilateral or bilateral, has no sex predilection, and is multifactoral. Patients with optic nerve hypoplasia have small, pale optic nerves with decreased numbers of optic nerve axons. Endocrine evaluation is warranted in these children to determine the presence of hypothalamic and pituitary dysfunction, which can result in growh hormone deficiency, neonatal , diabetes insipidus, panhypopituitarism, hyperprolactinemia, hypothyroidism, and (in rare instances) death. Neuroimaging is appropirate to assess for pituitary ectopia and structural midline defects. However, MRI would be preferred for assessing soft tissues rather than CT scanning. Although genetic testing my provide additonal information, it does not need to be ordered immediately. Similarly, photographs of the optic nerves are useful, but not required to make the diagnosis.

Question 100 of 130

A 6-year-old girl is diagnosed with anterior uveitis in her right eye. What is the preferred first-line therapy?

Prostiglandin inhibitors

Systemic immunosuppression

Observation

Topical corticosteroids

Please select an answer

Feedback: The goal of uveitis treatment in children is to suppress inflammation to prevent complications such as glaucoma, cataracts, retinal detachment, and amblyopia. Mild anterior segment inflammation can be treated with topical cortical steroids and cycloplegic agents. Oral corticosteroids and immunosuppressants should be given in conjunction with the child's rheumatoltogist or pediatrician in severe cases in which the risks outweigh the benefits. Postiglandin inhibitors are not currently considered preferred first-line therapy in children.

Question 101 of 130

Which of the following is considered an ocular complication of strabismus surgery?

Changes in refractive error

Cataract

Glaucoma

Cystoid Please select an answer Feedback: Multiple complications can be associated with strabismus surgery, eg, changes in refractive error, unsatisfactory alignment, diplopia, perforation of the sclera, lost or slipped muscle, anterior segment ischemia, fat adherence syndrome, and postoperative infection. Cataract, cystoid macular edema, and glaucoma are not recognized, reported complications following strabismus surgery.

Question 102 of 130

A 7-year-old patient is diagnosed with anisometropic amblyopia. Cycloplegic refraction is +2.00 OD, +7.00 OS. Which of the following is the most appropriate next step in treatment?

Patching or Atropine penalization

Bilateral refractive surgery

Close observation of vision

Optical correction with glasses Please select an answer Feedback: Anisometropic amblyopia develops in the presence of unequal refractive errors, resulting in chronic defocusing or image blur in one eye. Treatment includes correcting refractive errors initially, which can be followed as necessary by cycloplegia and forcing use of the poorer eye by limiting the use of the better eye. This is achieved with occlusion therapy and optical degradation (penalization) but must be done in combination with optical correction with glasses. While close observation of vision is an important part of amblyopia treatment, close observation of vision alone is not the most appropriate next step in treatment. Bilateral refractive surgery is not an appropriate treatment choice.

Question 103 of 130

Intraocular lens (IOL) implantation in children is relatively contraindicated in what setting?

Active inflammation/uveitis

Controlled glaucoma

Age less than 1 year

Previous ocular surgery Please select an answer Feedback: Placement of an IOL in an eye with active inflammation/uveitis is relatively contraindicated due to increased risk to visual and even ocular loss. Other contraindications for IOL implantation in children include microcornea, sclerocornea, microphthalmos, and uncontrolled glaucoma. Although the use of IOLs is considered investigational in children by the Food and Drug Administration, IOLs can be considered in children of all ages under appropriate circumstances.

Question 104 of 130

A 6-year-old girl is newly diagnosed with juvenile idiopathic arthritis (JIA). What condition is the most likely to develop in this patient?

Cell and flare in the posterior pole

Cell and flare in the anterior chamber

Endophthalmitis

Refractive error Please select an answer Feedback:

JIA is the most common cause of anterior uveitis in children and comprises a group of diseases characterized by chronic synovitis associated with extra-articular manifestations. The cause of JIA is unknown. The usual presentation is uveitis diagnosed after the onset of arthritis. Pertinent clinical features include cell and flare in the anterior chamber, synechiae, heterochromia, and joint pain. Disease-related complications include amblyopia, , cataract, glaucoma, and cystoid macular edema.

Question 105 of 130

Which of the following are risk factors for iridocyclitis in children with Juvenile idiopathic arthritis (JIA)?

Antinuclear antibodies (ANA) positive, rheumatoid factor negative

Antinuclear antibodies (ANA) positive, rheumatoid factor positive

Male gender and pauciarticular disease

Female gender and polyarticular disease Please select an answer Feedback:

JIA is the most common cause of anterior uveitis in children and comprises a group of diseases characterized by chronic synovitis associated with a extra-articular manifestations. The cause of JIA is unknown. The usual presentation is uveitis diagnosed after the onset of arthritis. Risk factors include antinuclear antibody positive (ANA+), rheumatoid factor negative children who are female with pauciarticular disease.

Question 106 of 130

A patient with esotropia is suspected of having an abnormal accommodative convergence/accommodation (AC/A) ratio. Which of the following methods should be used to measure the patient’s AC/A ratio?

Krimsky

Hirschberg

Gradient

Major amblyoscope Please select an answer Feedback:

Abnormalities of the AC/A ratio should be considered in patients with esotropia that is relatively greater at near than distance. The AC/A ratio can be measured using either the gradient or the method. The gradient method arrives at the AC/A ratio by the change in deviation in prism diopters divided by the change in lens power. An accommodative target is used, and the working distance is held constant. The heterophoria method arrives at the AC/A ratio by measuring the distance and near deviations. The Hirschberg, Krimsky, and major amblyoscope methods are corneal light-reflex tests useful in assessing ocular alignment in patients uncooperative with cover testing or having poor fixation.

Question 107 of 130

An infant boy presents with hyperacute purulent conjunctivitis. A diagnosis of Neisseria gonorrhoeae conjunctivitis is suspected. Which of the following is the most appropriate next step in the diagnosis?

Examine for preauricular adenopathy.

Corneal culture

Prompt conjunctival biopsy.

Gram stain and culture Please select an answer Feedback:

Bacterial conjunctivitis may present either hyperacutely or with the delayed onset of symptoms and signs. Bacterial conjunctivitis secondary to Neisseria species presents as a hyperacute (< 24 hours), purulent conjunctivitis. Gram stain and culture should be obtained. Preauricular adenopathy and corneal infiltrates can present in cases of bacterial conjunctivitis secondary to Neisseria. However, the findings are not specific for Niesseria conjunctivitis. Conjunctival biopsy is not required to make the diagnosis of bacterial conjunctivitis.

Question 108 of 130

Following discharge from a local hospital, a 6-day-old infant returns for evaluation and is diagnosed with ophthalmia neonatorum. Which of the following is the most likely cause of this patient’s condition?

Chlamydia trachomatis

Staphylococcus aureus

Herpes simplex

Niesseria gonorrhoeae Please select an answer Feedback:

Chlamydia trachomatis is the most common cause of ophthalmia neonatorum in the United States. The disease tends to present at 5 to 12 days of age, earlier when premature rupture of membranes occurs. Clinical features include mild swelling, hyperemia, and a papillary reaction associated with a watery discharge that is minimal to moderate. Copious discharge with pseudomembrane formation occur rarely. The diagnosis is established by conjunctival culture using Chlamydial transport medium, which shows intracytoplasmic inclusions by Giemsa stain. Direct fluorescent antibody tests and enzyme-linked immunoassays can also help establish the diagnosis. Treatment involves oral and topical erythromycin. Oral trimethoprim-sulfamethoxazole can be used as an alternative to oral erythromycin.

Question 109 of 130 A 3-year old girl is diagnosed with strabismic amblyopia. Following cycloplegic refraction, the child is also noted to have hyperopia equal to +5.00 sphere in each eye. Which of the following is the most appropriate next step in treatment?

Strabismus surgery

Cycloplegic agents

Full-time spectacles

Full-time occlusion Please select an answer Feedback:

Treatment of amblyopia includes correcting refractive errors, forcing use of the poorer eye by limiting the use of the better eye, and eliminating obstacles to vision (such as a cataract). Patients with amblyopia and significant refractive errors are given full-time spectacles to correct the full refractive error as determined with cycloplegia. Correction of refractive errors alone with full-time spectacles will improve vision in the amblyopic eye in some children. Forcing use of the poorer eye by limiting use of the better eye is achieved with occlusion therapy and optical degradation (penalization). Strabismus surgery should be considered after treatment of amblyopia.

Question 110 of 130

A 3-month-old infant undergoes lensectomy for a unilateral . Which of the following is most likely to develop in this patient?

Amblyopia

Vitreous hemorrhage

Secondary cataract

Hyphema

Please select an answer

Feedback: Children with unilateral cataracts are at particular risk for developing amblyopia, but children with bilateral cataracts are also at risk. Other disease-related complications associated with congenital cataracts include strabismus, glaucoma, and nystagmus. A primary posterior capsulotomy is usually created in young children undergoing cataract surgery. As a result, secondary cataracts (due to posterior capsule opacification) are not common.

Question 111 of 130

What relationship indicates that one pre-literate figure type (Allen cards, Lea symbols) is the preferred measure?

Allen pictures are more accurate than Lea figures.

Lea figures equate with Snellen acuity values.

Lea figures are more accurate than Allen pictures.

Allen pictures equate with Snellen acuity values. Please select an answer Feedback:

When evaluating vision, the most reliable test that the child can perform should be used. Snellen acuity is considered to be most reliable followed by HOTV, LEA symbols, the illiterate E test, Allen pictures, and fixation behavior. Preferential looking techniques with Teller acuity cards can be a useful adjunctive test for comparing visual acuity between fellow eyes in infants and preverbal children.

Question 112 of 130 What is the preferred management for anisometropic amblyopia?

Optical correction with glasses

Opaque contact lens over one eye Please select an answer Feedback:

Treatment of amblyopia involves the correction of significant refractive error, the elimination of obstacles to vision (such as cataract), and use of the poorer eye by limiting the use of the better eye. Patients with anisometropic amblyopia are treated with full-time spectacles to correct the full refractive error as determined with cycloplegia. Correction of refractive errors alone with full-time spectacles will improve vision in the amblyopic eye in some children. Forcing use of the poorer eye by limiting use of the better eye is achieved with occlusion therapy and optical degradation (pharmacological penalization). Occlusion therapy is achieved by covering the better eye with an adhesive patch or, rarely, opaque contact lens. Pharmacological penalization is usually achieved by blurring the vision of the better seeing eye with a cycloplegic agent, commonly Atropine. Observation without glasses, the use of pharmacological penalization alone, and the use occlusion therapy alone are not appropriate treatment options because these options do not correct significant refractive error.

Question 113 of 130

A 22-year-old patient with a compensatory head posture for V-pattern esotropia undergoes successful surgical realignment of the eyes. What is the patient most likely to report of the post-operative Worth 4-dot examination?

3 green lights – suppression of 1 eye

3 green lights, then 2 red lights – alternate suppression

5 lights – diplopia, but no suppression of either eye

Sees all 4 lights – normal response/fusion Please select an answer Feedback: The Worth 4-dot test is used to evaluate suppression and anomalous retinal correspondence. During testing, glasses are worn with a red lens over 1 eye and a green over the other. A target is shown at distance (20 feet) and near (33 cm). The target consists of a light source that has 1 red light, 2 green lights, and 1 white light. The patient is then asked to describe the number and color of lights. If all 4 lights are appreciated, the patient is considered to have fusion or a normal response. Following successful surgical realignment of the eyes in a patient with V-pattern esotropia, the patient should have a normal response during Worth 4-dot testing.

Question 114 of 130

The day following a motor vehicle accident, a 24-year-old man is diagnosed with unilateral superior oblique palsy. He has a small right hypertropia in primary gaze. The deviation is fairly comitant, with minor torsional component. What is the preferred intervention?

Prisms

Harada-Ito procedure (anterior half of superior oblique tendon advancement)

Superior oblique tuck

Orthoptic exercises Please select an answer Feedback:

Patients with unilateral superior oblique palsy can be managed with prisms if the deviation is small, fairly comitant, and with minor torsion. Surgical management is reserved for patients with large deviations that may also be incomitant and have significant torsion. Orthoptic exercises do not serve as an appropriate treatment choice for patients with superior oblique palsy.

Question 115 of 130

What type of tropia is a usually a V-pattern strabismus?

Sensory exotropia

Refractive esotropia

Infantile esotropia

Intermittent exotropia Please select an answer Feedback: V-pattern strabismus is a pattern in which the eyes are more abducted in downgaze than in upgaze. The etiology is unknown except in cases of extraocular muscle palsies. Multiple theories suggest primary or secondary muscle dysfunction, which can involve inferior oblique muscle overaction, superior oblique muscle underaction, horizontal rectus muscle dysfunction, or extorsion of the globe and bony which may lead to a V- or A- pattern. Infantile esotropia usually presents with a V-pattern. Refractive esotropia, intermittant exotropia and sensory exotropia are not commonly V-patterned.

Question 116 of 130

In what disease or post-operative setting is the 3-step test most helpful for diagnosing isolated cyclovertical muscle paralysis?

Hyperdeviations and previous oblique muscle surgery

Hyperdeviations and C-spine deformity

Hyperdeviations and restrictive strabismus

Hyperdeviations and previous cataract surgery Please select an answer Feedback:

The 3-step test is indicated in the diagnosis of isolated cyclovertical muscle palsies in patients with hyperdeviations. The test is contraindicated in patients with restrictive strabismus or previous strabismus surgery, especially of the cyclovertical extraocular muscles because the test can easily be misinterpreted. Care should also be used in interpreting the test if more than one cyclovertical muscle is involved. The test is also ineffective in patients with a history C-spine instability or fixed C-spin deformity due to limited patient participation during testing.

Question 117 of 130

When managing a vertical strabismus associated with thyroid , how should the timing of strabismus surgery be related to proptosis surgery?

Perform strabismus surgery before orbital decompression

Perform simultaneous strabismus surgery and eyelid surgery

Perform strabismus surgery after eyelid surgery

Perform strabismus surgery after orbital decompression Please select an answer Feedback:

In patients with thyroid eye disease, surgical management is indicated for diplopia or an anomalous head posture. Orbital decompression may adversely affect ocular motility and eyelid position. As a result, eyelid and extraocular muscle surgery should be deferred if orbital decompressive surgery is contemplated. Strabismus surgery may affect eyelid position. Thus, eyelid surgery should be deferred if extraocular muscle surgery is contemplated.

Question 118 of 130

Three days following strabismus surgery, a 6-year-old child develops increasing conjunctival injection and eyelid edema in one eye. The child has vision of 20/25 OU and his ocular motility is full. What is the most likely diagnosis?

Allergic conjunctivitis

Preseptal/orbital cellulitis

Anterior segment ischemia

Early postoperative injection Please select an answer Feedback:

Postoperative infections following strabismus surgery occur rarely, have typical onset 2 or 3 days postoperatively and can present as preseptal cellulitis, orbital cellulitis, or endophthalmitis. Early postoperative injection is not usually associated with increasing conjunctival injection or eyelid edema, which is described in this patient. The earliest signs of anterior segment ischemia are cells and flare in the anterior chamber. Corneal epithelial edema, folds in Descemet’s membrane, and an irregular pupil may characterize more severe cases. Findings consistent with anterior segment ischemia are not described in this patient. Likewise is not a likely diagnosis in this patient.

Question 119 of 130

A patient has strabismus surgery with adjustable sutures under using local anesthesia consisting of sub-Tenon's lidocaine injection. During postoperative adjustment, the patient experiences a vagal attack. Which of the following is the preferred management of this adverse event?

Discontinue suture adjustment and ensure adequate airway and circulation.

Continue suture adjustment after placing the patient in supine position.

Continue suture adjustment with the use of 100% oxygen.

Discontinue suture adjustment and reassess patient in 24 hours. Please select an answer Feedback:

Adjustable suture technique in strabismus surgery can be used in selected patients to adjust postoperative alignment. The technique is particularly useful in cases with unpredictable outcomes. Complications include movement during the critical part of the procedure in awake or inadequately anesthetized patients and lack of cooperation with postoperative suture adjustment. These complications can be minimized by ensuring adequate anesthesia and selecting patients carefully. If a patient experiences a vagal attack during postoperative suture adjustment, the procedure should be discontinued. In addition, the patient should be placed in a supine position, ensuring adequate airway and circulation.

Question 120 of 130

In what condition are cyclopegic agents contraindicated during refraction?

Infantile glaucoma

Adverse side effects to anticholinergic drugs

Young or uncooperative patients

Malingering

Please select an answer

Feedback:

The use of cycloplegic agents for cycloplegic refraction is an important part in the evaluation of patients with complaints pertaining to and ocular motility. Adverse reactions to cycloplegic agents (anticholinergics) include allergic or hypersensitive reaction with conjunctivitis, eyelid edema, and dermatitis. Psychological effects have been observed in children receiving the drug. Adverse reactions occur more frequently with atropine compared to other agents. Systemic intoxication from atropine manifests as dry mouth, fever, flushing of the face, rapid pulse, nausea, dizziness, delirium, and erythema. Patients with previous adverse side effects are at risk for adverse reactions to further use of the offending cycloplegic agents. In none of the other conditions are cycloplegic drops contraindicated.

Question 121 of 130

What is a risk factor for herpes zoster ophthalmicus in children?

Older age

Prior episode of chicken pox

Immunosuppression

Recent cold or flu Please select an answer Feedback:

Herpes zoster ophthalmicus is rare in children. Pertinent clinical features include vesicle formation on the head and eyelids and epiphora. Uveitis may also be present. Immunocompromised children are at greater risk for developing the disease. Older age, recent cold or flu, and prior episode of chicken pox are not considered risk factors for herpes ophthalmicus in children.

Question 122 of 130

A 3-day old infant is diagnosed with . What is the most likely means by which this child became infected?

Passage through the birth canal of an infected mother

Exposure to another infant with HSV-1 or HSV-2 infection

Exposure to infected

Exposure to a sibling infected with HSV-1 or HSV-2 Please select an answer Feedback:

Herpes simplex complex results from infection with herpes simplex viruses 1 and 2 (HSV-1 and HSV-2). Close personal contact with infected individuals is the primary risk factor for herpes simplex. Newborns with herpes simplex infection acquire the disease from passage through the infected birth canal of the mother. In such cases, HSV-2 is the usual agent. The diagnosis of herpes simplex is usually clinical, but virus can be cultured from vesicles.

Question 123 of 130

A 14-month-old boy is diagnosed with nasolacrimal duct obstruction. What is the preferred initial treatment?

Oral antibiotics

Topical steroids

Warm compresses

Digital massage Please select an answer Feedback:

Nasolacrimal duct obstruction most commonly results from a thin mucosal membrane at the lower end of the nasolacrimal duct (valve of Hasner). The incidence of nasolacrimal duct obstruction in newborns is 5-20%. The disorder has no sex predilection and has a high rate of spontaneous resolution. Spontaneous resolution becomes less likely beyond 1 year of age. Medical therapy options include digital massage, topical antibiotics, and observation. If this approach is unsuccessful, surgical therapy options include nasolacrimal duct probing, balloon catheter dilation, and silicone intubation. Nasolacrimal duct probing is more than 90% successful with few complications.

Question 124 of 130

What is a common ocular manifestation of herpes simplex complex infection in children?

Unilateral allergic conjunctivitis

Unilateral giant papillary conjunctivitis

Unilateral belpharoconjunctivitis

Unilateral vernal/atopic conjunctivitis Please select an answer Feedback:

Herpes simplex complex results from infection with herpes simplex viruses 1 and 2 (HSV-1 and HSV-2). Close personal contact with infected individuals is the primary risk factor for herpes simplex. The diagnosis of herpes simplex is usually clinical, but virus can be cultured from vesicles. Unilateral is considered to be a pertinent element in the history and manifestation of herpes simplex infection in children. Rarely the belpharitis can be bilateral. Patients may also present with watery discharge, decreased vision, vesicular lesions, and palpable preauricular lympadenopathy. Allergic, giant papillary and vernal conjunctivitis are usually bilateral and not typical of herpetic disease

Question 125 of 130

In what location does a dacryocele typically present as a bluish swelling?

Below and nasal to the medial canthus

Above the lateral canthal tendon

Above and lateral to the medial canthus

Below the lateral canthal tendon Please select an answer Feedback:

A dacryocele occurs when a distal blockage causes distention of the tear sac. Retrograde debries and tears causes a kink in the common canaliculus. When retrograde movement is blocked, decompression is prevented, resulting in the accumulation of secretions that stagnate in the tear sac. A dacryocele presents as a bluish swelling just below and nasal to the medial canthus. Dacryoceles typically bulge anteriorly below the medial canthal tendon and nasal to the canthus. A dacryocele may be associated with nasal obstruction (nasal mucocele), which can result in respiratory distress.

Question 126 of 130

A 7-year-old boy corneal pseudodendrites from herpes zoster ophthalmicus. What is the preferred management of this patient?

Topical antiviral medication

Topical corticosteroids and antibiotics

Systemic corticosteroids and antibiotics

Systemic antiviral medication

Please select an answer Feedback:

Herpes zoster ophthalmicus is rare in children. Pertinent clinical features include epiphora and vesicle formation on the head and eyelids. Uveitis may also be present. Immunocompromised children are at greater risk for developing the disease. Management includes systemic antivirals (eg, acyclovir, famciclovir, valacyclovir), moist compresses, topical antibiotics for cutaneous lesions, and topical lubricants. Topical corticosteroids and cycloplegics are used for stromal or disciform keratitis or uveitis, not features mentioned for this child.

Question 127 of 130

A 4-year-old girl is diagnosed with orbital cellulitis. What is the preferred management?

Hospitalize and start broad-spectrum intravenous antibiotics

Start oral antibiotics and evaluate visual acuity weekly as an outpatient

Start oral antibiotics and evaluate visual acuity daily as an outpatient

Hospitalize and closely evaluate vision and ocular motility Please select an answer Feedback:

It is crucial to distinguish orbital cellulitis from preseptal cellulitis because the former requires hospitalization and treatment with IV broad-spectrum antibiotics along with close evaluation of vision and ocular motility. Orbital cellulitis may occur at any age and may occur in patients with preceding sinusitis. Patients commonly have a history of periocular pain or headache, swelling and tenderness of the eyelids, fever, and lethargy. Decreased vision and diplopia are also possible. Computed tomography or magnetic resonance imaging of the orbits and sinuses can help establish the diagnosis.

Question 128 of 130

A 4-year-old child develops post-traumatic uveitis in the right eye. The child is started on atropine eyedrops. What condition is a potential complication of this therapy?

posterior iris synechiae in the right eye

solar retinopathy in the right eye

Confusion

systemic immunosuppression Please select an answer Feedback:

Confusion is one manifestation of a systemic anticholinergic side effect and may be dose-related. Bradycardia, hyperthermia, generalized flushing, and even pharmacologic delirium may result from anticholinergic side effects. Solar retinopathy results from prolonged viewing of the sun, which is not likely in the setting of hyperactivity from anticholinergic overdose. Posterior synechiae may result from post-traumatic uveitis, but not from atropine. Atropine does not have an immunosuppressive effect.

Question 129 of 130

A 13-year-old boy has blurry vision and diplopia when reading. He has a remote near-point of convergence. On alternate cover testing he has no distance deviation, but at near has an exophoria. What is his most likely diagnosis?

Intermittent exotropia, simulated divergence excess type

Convergence insufficiency

Intermittent exotropia, convergence insufficiency type

Accommodative insufficiency Please select an answer Feedback:

Common symptoms of convergence insufficiency include asthenopia, reading problems, blurred near vision, and diplopia. The sensorimotor examination shows an exophoria at near with no significant distance deviation. A remote near point of convergence is also present on examination. Accommodative insufficiency is not present in patients with convergence insufficiency.Unlike convergence insufficiency, intermittent exotropia is a divergent strabismus in which one eye appears to deviate out at times. Individuals with intermittent exotropia do not have exophoria alone on sensorimotor examination.

Question 130 of 130

Using adjustable sutures for strabismus surgery is preferred for patients in what setting?

Very young patients with unsure preoperative measurements

Reoperative patients with unpredictable surgical outcomes

Patients with episodic anxiety

Patients with frequent prior vagal attacks

Please select an answer

Feedback: The adjustable suture technique for strabismus surgery is usually employed to increase the likelihood of success with a single operation in patients with unpredictable outcomes, such as those undergoing reoperations. Despite this, the adjustable suture technique alone does not ensure long-term satisfactory alignment. During the adjustable suture technique, surgery is completed using externalized sutures and slip knots so that the position of the muscle can be altered during the early postoperative days or within only a few hours. The adjustable suture technique can be used successfully in children, but usually requires additional general anesthesia. Anxiety and known prior vagal episodes are relative contraindications to adjustable suture use

Collected from www.aao.org (self assessment)

By Dr. AlBaraa AlQassimi